The question bank may take some time to load… Just enough time to stretch, blink a few times, and question your life choices — but not too long, we promise!
We recommend going Full Screen for the best experience. Have Fun !
Report a question
NEUROSCIENCE – 2022
Questions from the 2022 Neuroscience Module + Annuals
Initially, the reflexes might be sluggish or absent, but over time, they may become overactive due to the disruption of higher brain control.
1 / 118
A “crossed pattern” of motor deficits (one side of the face, opposite side of the body) often means a brainstem lesion —where does the hypoglossal nerve originate ?
2 / 118
Category:
NeuroSciences – Anatomy
A 55-year-old man presents to the neurology clinic with the complaint of right-sided weakness of the body and left-sided weakness of the face with the tongue deviating to the left upon protrusion. Which of the following structures is most likely to be affected in this case?
This patient presents with a classic “crossed signs” neurological deficit :
✅ Right-sided hemiparesis (body weakness) → Suggests an issue affecting descending motor pathways (corticospinal tract).
✅ Left-sided facial weakness → Indicates involvement of the cranial nerves , particularly the hypoglossal nerve (CN XII) .
✅ Tongue deviates to the left → Suggests left hypoglossal nerve (CN XII) dysfunction , which originates in the medulla .
Localization of the Lesion:
This is characteristic of a medullary lesion , most likely Medial Medullary Syndrome (Dejerine Syndrome) , caused by an anterior spinal artery infarction affecting the:
Corticospinal tract (contralateral hemiparesis).
Medial lemniscus (contralateral loss of proprioception and fine touch—may not be mentioned here).
Hypoglossal nerve (CN XII) nucleus and fibers (ipsilateral tongue weakness, deviation toward the lesion).
Why not the other options?
Pons → Would involve CN VII (facial nerve) , causing ipsilateral full facial weakness rather than tongue deviation .
Midbrain → Would affect CN III (oculomotor nerve) , causing eye movement deficits (e.g., Weber’s syndrome).
Cerebellum → Causes ataxia, intention tremor, and coordination deficits , not motor weakness or tongue deviation.
Cerebral cortex → Would cause contralateral hemiparesis of face and body but not crossed signs (i.e., right body + left cranial nerve deficits).
Thus, the correct answer is medulla , as the symptoms strongly suggest Medial Medullary Syndrome , affecting the corticospinal tract, medial lemniscus, and hypoglossal nerve (CN XII) .
When the main bridge between brain hemispheres is missing , fluid-filled spaces expand , creating an abnormally enlarged and spaced ventricle appearance on imaging.
3 / 118
Category:
Neurosciences – Embryology
A child is born with agenesis of the corpus callosum which is a major commissural fiber. Which of the following findings is most likely to be seen in radiological scans of this child’s brain?
Agenesis of the corpus callosum disrupts the normal development of commissural fibers , leading to abnormal lateral ventricle morphology , particularly:
✅ Colpocephaly → Enlargement of the posterior (occipital) horns of the lateral ventricles , giving them a characteristic “teardrop” or “bat-wing” appearance.
✅ Widely spaced lateral ventricles → Due to the absence of the corpus callosum , the ventricles appear farther apart, forming the “racing car sign” on axial MRI/CT scans.
✅ Interhemispheric cysts → Sometimes present, further contributing to ventricular abnormalities.
Why not “Prominent Forceps Major” as the Best Answer?
The forceps major (posterior part of the corpus callosum) is absent in ACC, leading to abnormal Probst bundles , but its prominence is not as striking as colpocephaly in imaging studies.
Colpocephaly is the hallmark and most striking radiological feature of ACC .
Thus, “Deformed lateral ventricles (Colpocephaly)” is the best answer , as it is the most prominent and defining radiological finding in agenesis of the corpus callosum.
This cerebellar region talks to the motor cortex to plan movements before they happen , ensuring smooth execution.
4 / 118
Category:
NeuroSciences – Anatomy
Which of the following parts of the cerebellum is involved in planning and coordinating movements?
The lateral zone (cerebrocerebellum) and the dentate nucleus of the cerebellum are responsible for planning and coordinating movements .
Key Functions of the Lateral Zone and Dentate Nucleus:
✅ Cerebrocerebellum (Lateral Hemisphere)
Involved in motor planning, coordination of voluntary movements, and fine motor control .
Receives input from the cerebral cortex via the pontine nuclei .
Sends output to the dentate nucleus , which projects to the motor cortex via the thalamus .
✅ Dentate Nucleus
Largest of the deep cerebellar nuclei .
Plays a critical role in initiating and planning voluntary movements .
Why not the other options?
Intermediate zone and interposed nuclei → Involved in ongoing movement adjustments (limb coordination), not motor planning.
Vermis and interposed nuclei → The vermis controls axial muscle coordination , not motor planning.
Vermis and fastigial nucleus → The fastigial nucleus is involved in posture and balance , not planning movements.
Lateral zone and fastigial nucleus → The fastigial nucleus is associated with balance and eye movements , not movement planning.
Thus, the correct answer is “Lateral zone and dentate nucleus,” as these structures are responsible for motor planning and coordination .
This brainstem region is a natural painkiller hub —when damaged, pain perception increases due to loss of inhibition.
5 / 118
This natural anticoagulant inhibits thrombin and is supercharged by heparin to prevent clotting.
6 / 118
Category:
Neurosciences – Pharmacology
Heparin binds to which of the following molecules to produce its anti-coagulant effect?
Heparin exerts its anticoagulant effect by binding to and enhancing the activity of anti-thrombin III (AT-III) .
Anti-thrombin III is a serine protease inhibitor (serpin) that inactivates clotting factors (mainly thrombin (Factor IIa) and Factor Xa ).
Heparin binding increases AT-III activity by 1000-fold , leading to rapid inhibition of thrombin and Factor Xa , preventing clot formation.
Mechanism of Action of Heparin:
✅ Heparin binds to anti-thrombin III , causing a conformational change that accelerates its ability to inhibit thrombin and Factor Xa .
✅ This prevents fibrin formation , reducing blood clotting.
✅ Low molecular weight heparin (LMWH) primarily inhibits Factor Xa rather than thrombin.
Why not the other options?
Protein C → Involved in anticoagulation , but it is activated by thrombomodulin-bound thrombin , not heparin.
Vitamin K → Needed for the synthesis of coagulation factors (II, VII, IX, X) , but heparin does not interact with it (warfarin affects vitamin K-dependent factors).
Von Willebrand factor (vWF) → Important in platelet adhesion , but heparin does not act on vWF.
Fibrinogen → Converted into fibrin by thrombin to form clots, but heparin acts upstream by inhibiting thrombin formation .
Thus, the correct answer is anti-thrombin III , as heparin enhances its anticoagulant activity to prevent blood clot formation.
This ATP-powered transporter keeps many drugs and toxins out of the brain , making it a major challenge for CNS drug delivery .
7 / 118
Category:
Neurosciences – Biochemistry
Which of the following is an active efflux transporter in the blood-brain barrier?
P-glycoprotein (P-gp) is an active efflux transporter located in the blood-brain barrier (BBB) . It functions to pump out potentially harmful substances, including drugs, from the brain back into the bloodstream using ATP-dependent transport .
Key Features of P-glycoprotein:
✅ Acts as a protective mechanism → Prevents toxins and certain drugs from accumulating in the brain.
✅ Energy-dependent efflux transporter → Uses ATP to actively transport substances out of endothelial cells .
✅ Limits drug penetration into the CNS → Contributes to drug resistance in conditions like epilepsy and chemotherapy-resistant brain tumors.
✅ Encoded by the MDR1 (ABCB1) gene .
Why not the other options?
GLUT-1 → A passive glucose transporter in the BBB, responsible for glucose uptake into the brain .
GLUT-3 → A high-affinity glucose transporter found in neurons , involved in glucose metabolism , not drug efflux.
GLUT-4 → An insulin-dependent glucose transporter found in muscle and adipose tissue , not the BBB.
SGLT-2 → A sodium-glucose cotransporter in the renal proximal tubules , involved in glucose reabsorption in the kidneys , not the BBB.
Thus, the correct answer is P-glycoprotein , as it is the active efflux transporter that prevents unwanted substances from crossing into the brain .
This brain region helps convert sounds into meaningful words —without it, speech sounds like meaningless noise .
8 / 118
Category:
NeuroSciences – Anatomy
A 56-year-old woman presents to the outpatient clinic with the complaint of not being able to understand spoken language. She can, however, interpret written symbols and speech. Which of the following areas of the cerebral cortex is most likely to be damaged in this case?
This 56-year-old woman presents with:
✅ Inability to understand spoken language → Suggests a defect in auditory processing .
✅ Can interpret written symbols and speech → Suggests that comprehension of written language is intact , ruling out Wernicke’s area damage .
This condition is most likely due to damage to the secondary auditory area (Brodmann areas 42 and 22), which is involved in higher-order auditory processing and speech perception .
Key Role of the Secondary Auditory Cortex:
Located in the superior temporal gyrus .
Responsible for processing complex auditory signals .
Converts sounds into meaningful language perception .
Damage leads to auditory agnosia , where the patient hears sounds but cannot recognize them as words.
Why not the other options?
Wernicke’s area → Damage causes Wernicke’s aphasia , where patients cannot understand spoken or written language , but this patient can interpret written symbols .
Primary visual area (Brodmann area 17) → Processes visual input , but does not directly relate to spoken language comprehension.
Primary auditory area (Brodmann area 41) → Detects basic sounds and pitch , but does not interpret meaning .
Secondary visual area → Involved in visual processing , not auditory comprehension.
Thus, the correct answer is secondary auditory area , as it plays a crucial role in understanding spoken language .
This white matter tract allows you to repeat what you hear —damage to it results in conduction aphasia .
9 / 118
This nervous system is responsible for the “fight or flight” response and has a chain of ganglia running along the spine .
10 / 118
Category:
Neurosciences – Physiology
Which part of the nervous system consists of two ganglionated nerve trunks extending the entire length of the vertebral column on both sides?
The sympathetic nervous system (SNS) consists of two ganglionated nerve trunks , known as the sympathetic chains (trunks) , that extend bilaterally along the entire length of the vertebral column .
Each sympathetic chain is made up of:
✅ Paravertebral ganglia → Linked to form a chain of ganglia running from the cervical to the sacral region .
✅ Prevertebral (collateral) ganglia → Located near major abdominal arteries (e.g., celiac ganglion).
✅ Postganglionic fibers → Travel to target organs (e.g., heart, lungs, blood vessels).
Functions of the Sympathetic Nervous System:
Activates the “fight or flight” response.
Increases heart rate, blood pressure, and respiration .
Dilates pupils and inhibits digestive functions .
Why not the other options?
Parasympathetic nervous system → Does not have a continuous chain of ganglia ; instead, it has ganglia near target organs (e.g., ciliary ganglion, otic ganglion).
Somatic nervous system → Controls voluntary muscle movements and lacks ganglia.
Enteric nervous system → Controls the gastrointestinal tract and has localized ganglia (not a continuous chain).
Autonomic nervous system → Includes both sympathetic and parasympathetic systems , but only the sympathetic system has the paired ganglionated nerve trunks .
Thus, the correct answer is sympathetic nervous system , as it consists of two ganglionated nerve trunks running parallel to the vertebral column .
This thin dural membrane covers the pituitary gland and prevents direct contact with the optic chiasm , allowing the pituitary stalk to pass through it.
11 / 118
Category:
NeuroSciences – Anatomy
While operating on the pituitary gland, a surgeon tries to secure the optic chiasma. Which of the following structures separates the pituitary gland from the optic chiasm?
The diaphragmatic sellae is a thin dural fold that separates the pituitary gland from the optic chiasm . It is part of the dura mater and covers the sella turcica , leaving a small opening for the passage of the pituitary stalk (infundibulum) .
Anatomical Relationship:
Pituitary gland lies within the sella turcica of the sphenoid bone.
Optic chiasm is located superior to the diaphragmatic sellae .
The diaphragmatic sellae acts as a roof over the pituitary fossa , preventing direct compression of the gland against the chiasm.
Why not the other options?
Dorsum sellae → The posterior wall of the sella turcica , but it does not separate the pituitary gland from the optic chiasm.
Tuberculum sellae → The anterior wall of the sella turcica, which provides attachment for the diaphragmatic sellae but does not directly separate the structures.
Cavernous sinus → Located laterally to the pituitary gland, containing cranial nerves III, IV, V1, V2, and VI , but does not separate the gland from the optic chiasm.
Sphenoidal sinus → Located below the sella turcica , but it has no role in separating the pituitary from the optic chiasm.
Thus, the correct answer is diaphragmatic sellae , as it forms a protective dural layer between the pituitary gland and the optic chiasm .
Once paralysis has set in, prevention is no longer an option —what’s the next best step to help the child function better ?
12 / 118
Category:
Neurosciences – Community Medicine + Behavioural Sciences
Which of the following is the most appropriate intervention after a child has been diagnosed with paralytic poliomyelitis?
Once a child is diagnosed with paralytic poliomyelitis , the virus has already caused irreversible motor neuron damage , leading to muscle weakness and paralysis . At this stage, the most appropriate intervention is rehabilitation , which aims to maximize functional recovery and improve quality of life .
Key Aspects of Rehabilitation for Poliomyelitis:
✅ Physical therapy → Prevents muscle atrophy and contractures.
✅ Occupational therapy → Helps the child adapt to daily activities.
✅ Orthotic devices and mobility aids → Braces, crutches, or wheelchairs to assist movement.
✅ Surgical interventions (if needed) → For deformities or contractures.
Why not the other options?
Awareness → Important for prevention , but it does not help an already paralyzed child.
Vaccination → Prevents poliomyelitis but does not reverse paralysis once the child is infected.
Nutritional supplementation → May support general health but does not treat paralysis .
Health education → Useful for prevention and understanding of the disease , but not the primary intervention after diagnosis.
Thus, the most appropriate intervention after poliomyelitis has occurred is rehabilitation , as it helps the child regain function and mobility.
This toxin is actively secreted by Corynebacterium diphtheriae and inhibits protein synthesis , leading to cell death and tissue damage .
13 / 118
Category:
Neurosciences – Community Medicine + Behavioural Sciences
Which of the following terms correctly describes the diphtheria toxin?
The diphtheria toxin , produced by Corynebacterium diphtheriae , is classified as an exotoxin because it is a secreted protein toxin that disrupts host cell function.
Key Features of Diphtheria Toxin:
✅ Secreted by bacteria → Unlike endotoxins, which are part of the bacterial cell wall.
✅ A-B toxin structure :
A (active) subunit → Inhibits protein synthesis by ADP-ribosylating elongation factor-2 (EF-2) , leading to cell death.
B (binding) subunit → Allows toxin entry into host cells.
✅ Causes pseudomembrane formation in the throat , myocarditis, and neuropathy.
Why not the other options?
Neurotoxin → Affects the nervous system (e.g., botulinum toxin, tetanus toxin), but diphtheria toxin mainly damages epithelial and cardiac cells .
Endotoxin → Found in the outer membrane of Gram-negative bacteria (e.g., lipopolysaccharide or LPS), whereas diphtheria toxin is a secreted protein .
Enterotoxin → Affects the gastrointestinal tract , causing diarrhea (e.g., cholera toxin, shiga toxin), but diphtheria does not cause GI symptoms.
Cytotoxin → While diphtheria toxin damages cells, the correct classification is exotoxin , since it is secreted.
Thus, the correct answer is “Exotoxin,” as diphtheria toxin is a secreted bacterial protein that disrupts host cell function .
This part of the brain looks white because it’s packed with myelinated nerve fibers , helping signals travel fast .
14 / 118
Category:
Neurosciences – Histology
Which of the following cell structures constitute the white matter of the central nervous system?
The white matter of the central nervous system (CNS) is primarily composed of:
✅ Myelinated axons → These transmit nerve signals over long distances.
✅ Glial cells → Mainly oligodendrocytes , which produce myelin, and astrocytes , which support neuronal function.
White matter appears white due to the high lipid content of myelin , which insulates axons and speeds up electrical transmission.
White Matter vs. Gray Matter:
White matter → Contains myelinated axons and glial cells but few or no cell bodies .
Gray matter → Contains neuronal cell bodies, dendrites, and synapses (e.g., cerebral cortex, basal ganglia).
Why not the other options?
Cell bodies only → Cell bodies are found in gray matter, not white matter .
Axons and dendrites → Dendrites are mostly in gray matter , where synapses occur.
Cell bodies and dendrites → This describes gray matter , not white matter.
Axons only → White matter also contains glial cells , particularly oligodendrocytes.
Thus, the correct answer is “Axons and glial cells,” as these are the main components of white matter in the CNS.
Imagine suddenly falling asleep in the middle of a conversation or collapsing when laughing —which disorder fits this?
15 / 118
Category:
Neurosciences – Physiology
Which of the following disorders is characterized by excessive daytime sleepiness and sudden loss of muscle tone?
Narcolepsy is a neurological sleep disorder characterized by:
✅ Excessive daytime sleepiness (EDS) → Sudden, uncontrollable sleep episodes.
✅ Cataplexy → Sudden loss of muscle tone triggered by strong emotions (e.g., laughter, surprise).
✅ Sleep paralysis → Temporary inability to move when falling asleep or waking up.
✅ Hypnagogic/hypnopompic hallucinations → Vivid dream-like experiences at sleep onset or upon waking.
Cause of Narcolepsy:
Linked to hypocretin (orexin) deficiency in the lateral hypothalamus , which regulates wakefulness.
Why not the other options?
Epilepsy → Causes seizures , not excessive sleepiness or cataplexy.
Somnolence → Refers to general drowsiness , but lacks the specific features of narcolepsy.
Cataplexy → A symptom of narcolepsy, but narcolepsy includes more than just muscle weakness.
Alzheimer’s disease → Causes cognitive decline and memory loss , not sudden sleep episodes or muscle tone loss.
Thus, the correct answer is Narcolepsy , as it is the disorder characterized by excessive daytime sleepiness and cataplexy .
This nerve lets you shrug your shoulders and turn your head —without it, even a simple “no” shake becomes difficult .
16 / 118
This area helps you speak fluently , but if damaged, you struggle to form words despite understanding speech .
17 / 118
Category:
NeuroSciences – Anatomy
Which of the following Brodmann’s areas corresponds to the motor speech area of Broca?
Brodmann area 44 , along with Brodmann area 45 , corresponds to Broca’s area , which is responsible for motor speech production .
Location : Found in the inferior frontal gyrus of the dominant hemisphere (usually the left).
Function : Controls the motor aspects of speech , including articulation and fluency.
Lesion in Broca’s area : Causes Broca’s (expressive) aphasia , characterized by:
✅ Non-fluent, effortful speech
✅ Intact comprehension
✅ Difficulty forming complete sentences
Why not the other options?
Brodmann area 7 → Located in the parietal lobe , involved in somatosensory association and spatial processing .
Brodmann area 17 → The primary visual cortex , responsible for processing visual information .
Brodmann area 22 → Includes Wernicke’s area , responsible for speech comprehension , not speech production.
Brodmann area 41 → The primary auditory cortex , involved in processing sound , not speech production.
Thus, the correct answer is Brodmann area 44 , as it is a key component of Broca’s motor speech area .
This opioid drug helps stop excessive bowel movements but doesn’t get you high —what is it?
18 / 118
Category:
Neurosciences – Pharmacology
Which of the following opioid drugs is used to treat diarrhea?
Loperamide is an opioid agonist that acts on mu (μ) opioid receptors in the gastrointestinal (GI) tract to slow intestinal motility , making it effective for treating diarrhea .
Unlike other opioids, loperamide has minimal central nervous system (CNS) effects because it does not cross the blood-brain barrier in significant amounts .
It reduces peristalsis , allowing for increased fluid absorption and stool solidification .
It is commonly used for acute and chronic diarrhea (e.g., irritable bowel syndrome, traveler’s diarrhea, gastroenteritis) .
Why not the other options?
Levorphanol → A strong opioid analgesic used for pain management, not diarrhea.
Morphine → A potent opioid pain reliever , but its main effect is analgesia, not GI motility reduction .
Naloxone → An opioid antagonist used to reverse opioid overdose , not for treating diarrhea.
Methadone → Used for opioid withdrawal and chronic pain , not diarrhea treatment.
Thus, the correct answer is loperamide , as it is specifically designed for treating diarrhea by reducing GI motility without significant CNS effects.
Damage to this compact white matter structure leads to contralateral weakness with increased reflexes and tone —what kind of paralysis does that indicate?
19 / 118
Category:
Neurosciences – Pathology
A 65-year-old male suffers from a cerebrovascular accident. An MRI scan reveals a lesion in the left internal capsule. Which of the following signs or symptoms is most likely to be observed in this patient?
A lesion in the left internal capsule affects the descending corticospinal tract , which carries motor commands from the primary motor cortex (precentral gyrus) to the contralateral body . Since motor control is contralateral , a left-sided internal capsule lesion will result in right-sided motor deficits .
Key Features of an Internal Capsule Stroke:
✅ Contralateral spastic paralysis (upper motor neuron lesion) → Damage to the corticospinal tract leads to increased muscle tone (spasticity) and hyperreflexia .
✅ Contralateral motor deficits → Since the corticospinal tract decussates at the medulla, a left-sided lesion affects the right side of the body.
✅ Can involve both upper and lower limbs → The posterior limb of the internal capsule carries motor fibers for the entire contralateral body.
Why not the other options?
Flaccid paralysis in left leg → Incorrect because internal capsule lesions cause UMN (spastic) paralysis, not LMN (flaccid) paralysis , and the affected side would be contralateral, not ipsilateral .
Flaccid paralysis in right leg → Internal capsule lesions cause spastic, not flaccid paralysis . Flaccid paralysis is seen in LMN lesions (e.g., peripheral nerve injury, anterior horn cell damage) .
Hyporeflexia in left leg → Hyporeflexia occurs in LMN lesions , while internal capsule lesions cause hyperreflexia (UMN signs) , and the affected side would be contralateral (right leg), not ipsilateral (left leg) .
Spastic paralysis in left leg → Incorrect because a left-sided internal capsule lesion affects the contralateral (right) side of the body .
Thus, the correct answer is spastic paralysis in the right leg , as UMN lesions in the left internal capsule affect the contralateral body .
The CSF is being produced but cannot flow past a blocked passageway —which narrow channel in the brain is most likely affected?
20 / 118
Category:
Neurosciences – Pathology
A child suffers from non-communicating hydrocephalus. Which of the following is a characteristic of non-communicating hydrocephalus?
Non-communicating (obstructive) hydrocephalus occurs when CSF flow is blocked within the ventricular system , preventing it from reaching the subarachnoid space . One of the most common sites of obstruction is the cerebral aqueduct (Aqueduct of Sylvius) , which connects the third and fourth ventricles .
Key Features of Non-Communicating Hydrocephalus:
✅ Obstruction within the ventricular system → Prevents CSF from circulating properly.
✅ Ventricular dilation proximal to the obstruction → Leads to increased intracranial pressure (ICP) , causing symptoms such as headache, vomiting, and papilledema .
✅ Common causes → Congenital aqueductal stenosis, tumors (e.g., medulloblastoma, ependymoma), Arnold-Chiari malformations .
Why not the other options?
Post-hemorrhagic → More commonly associated with communicating hydrocephalus , where CSF resorption is impaired due to blood-induced scarring of arachnoid granulations .
Decreased absorption of CSF → Seen in communicating hydrocephalus , where the arachnoid granulations fail to absorb CSF (e.g., after meningitis or hemorrhage).
Post-infectious → More associated with communicating hydrocephalus , often due to meningitis causing scarring of the arachnoid granulations .
Increased absorption of CSF → Does not cause hydrocephalus; hydrocephalus is due to excess CSF accumulation , not increased absorption.
Thus, the correct answer is “Blockage of cerebral aqueduct,” as it is a classic cause of non-communicating hydrocephalus .
The brain needs nutrients but must also be protected from toxins, infections, and fluctuations in blood chemistry —what structure manages this balance?
21 / 118
Category:
Neurosciences – Biochemistry
Which of the following statements correctly describes the main function of the blood-brain barrier?
The blood-brain barrier (BBB) is a highly selective barrier that:
✅ Regulates the entry of substances into the central nervous system (CNS) .
✅ Prevents toxins, pathogens, and harmful chemicals from reaching the brain .
✅ Allows essential nutrients (glucose, amino acids, oxygen) to pass through .
Key Features of the BBB:
Formed by tight junctions between endothelial cells of brain capillaries .
Supported by astrocyte foot processes and pericytes , which help regulate permeability.
Uses selective transport mechanisms for nutrients and essential molecules.
Why not the other options?
Detects changes in the chemical composition of blood → This is the role of circumventricular organs (e.g., area postrema, subfornical organ), not the BBB .
Secretes neurotransmitters → Neurons , not the BBB, secrete neurotransmitters.
Acts as a shock absorber → This is the function of cerebrospinal fluid (CSF), not the BBB .
Secretes cerebrospinal fluid (CSF) → CSF is secreted by the choroid plexus in the ventricles , not the BBB.
Thus, the correct answer is “Supplies nutrition to and prevents hazardous substances from reaching the brain,” as this describes the protective and regulatory role of the blood-brain barrier .
In Parkinson’s, the resting tremor appears on one side , but the damage is on the opposite side of the brain . What part of the brain degenerates in PD?
22 / 118
Category:
NeuroSciences – Anatomy
A 55-year-old man with a history of resting tremors in his right hand dies in a road traffic accident and undergoes an autopsy. His neurological examination is likely to reveal degeneration of neurons in which of the following structures?
The patient’s history of resting tremors suggests Parkinson’s disease (PD) , which is characterized by degeneration of dopaminergic neurons in the substantia nigra pars compacta .
Neurons in the substantia nigra produce dopamine , which modulates the basal ganglia circuits controlling movement.
Loss of dopamine leads to resting tremors, bradykinesia, rigidity, and postural instability .
Why the lesion is in the left substantia nigra?
The substantia nigra projects to the contralateral motor cortex via the basal ganglia-thalamocortical circuit.
Since right hand tremors are observed, the lesion is in the left substantia nigra (contralateral control).
Why not the other options?
Left corpus striatum → Includes the caudate and putamen , which are affected in Huntington’s disease , not Parkinson’s.
Left globus pallidus → Involved in motor regulation but not the primary site of degeneration in Parkinson’s.
Right globus pallidus → Parkinson’s is due to substantia nigra degeneration, not the globus pallidus.
Right substantia nigra → Would affect the left side, but the patient has right-sided tremors , meaning the lesion is in the left substantia nigra .
Thus, the correct answer is left substantia nigra , as dopaminergic neuron loss in the substantia nigra pars compacta leads to Parkinson’s disease symptoms contralaterally .
Which neurotransmitter is released in a fight-or-flight response and affects both vasoconstriction and heart rate ?
23 / 118
Category:
Neurosciences – Physiology
Which of the following neurotransmitters equally excites both alpha and beta-adrenergic receptors?
Epinephrine is the primary neurotransmitter that equally excites both alpha (α) and beta (β) adrenergic receptors . It is released by the adrenal medulla and plays a key role in the “fight or flight” response .
Epinephrine’s Effects on Adrenergic Receptors:
Alpha (α1, α2) effects → Vasoconstriction, increased blood pressure, pupil dilation (mydriasis) .
Beta (β1, β2, β3) effects → Increased heart rate (β1), bronchodilation (β2), lipolysis (β3) .
At low doses → Preferentially stimulates β receptors (vasodilation, bronchodilation).
At high doses → α receptors dominate , causing vasoconstriction and increased blood pressure .
Why not the other options?
Tyrosine → A precursor to catecholamines (dopamine, norepinephrine, epinephrine), but not a direct neurotransmitter.
Acetylcholine → Activates cholinergic (muscarinic and nicotinic) receptors , not adrenergic receptors.
Dopamine → Primarily activates dopaminergic (D1-D5) receptors and at higher doses stimulates β1 and α1 , but does not equally excite both α and β receptors like epinephrine.
Norepinephrine → Primarily stimulates α1, α2, and β1 but has very weak β2 effects , making it less balanced than epinephrine.
Thus, epinephrine is the correct answer , as it equally excites both α and β adrenergic receptors .
When someone overdoses on opioids , this drug kicks opioids off their receptors and brings them back to life .
24 / 118
Category:
Neurosciences – Pharmacology
Which of the following is an opioid receptor antagonist used as a drug for opioid overdose and toxicity?
Naloxone is a competitive opioid receptor antagonist that rapidly reverses opioid overdose and toxicity by blocking mu (μ), kappa (κ), and delta (δ) opioid receptors .
Key Features of Naloxone:
✅ Used in opioid overdose to reverse respiratory depression, sedation, and miosis .
✅ Has a short half-life (~1 hour) , so repeated doses may be required for long-acting opioids.
✅ Administered IV, IM, SC, or intranasally for rapid onset.
Why not the other options?
Nalbuphine → A mixed opioid agonist-antagonist (κ agonist, μ antagonist ), but not used for opioid overdose .
Buprenorphine → A partial μ-opioid agonist , used for opioid addiction treatment , not overdose reversal.
Morphine → A full opioid agonist , causes opioid toxicity , rather than reversing it.
Fentanyl → A potent synthetic opioid agonist , responsible for many overdoses, rather than treating them.
Thus, the correct answer is Naloxone , as it is the gold standard opioid antagonist for overdose reversal .
These fibers are responsible for sensing muscle tension and help prevent muscle injury from overexertion . What type of sensory afferent would handle this task?
25 / 118
Category:
Neurosciences – Physiology
Which of the following types of nerve fibers innervate Golgi tendon organs?
Golgi tendon organs (GTOs) are sensory receptors located at the junction of muscles and tendons. They are primarily involved in monitoring muscle tension and protecting muscles from excessive force.
Ib sensory afferents innervate Golgi tendon organs .
When tension increases within the muscle, Ib fibers are activated , sending signals to the spinal cord.
This triggers the inverse myotatic reflex (autogenic inhibition) , causing muscle relaxation to prevent tendon damage.
Function of Golgi Tendon Organs:
Detect changes in muscle tension .
Protect muscles from excessive force by inhibiting alpha motor neurons through inhibitory interneurons.
Important for fine-tuning motor activity and maintaining posture .
Why not the other options?
II sensory afferents → Associated with static stretch receptors in muscle spindles , not GTOs.
III sensory afferents → Involved in nociception (pain) and temperature , not muscle tension detection.
IV sensory afferents → Unmyelinated fibers that carry pain and temperature , not proprioceptive information.
Ia sensory afferents → Innervate muscle spindles , specifically responding to dynamic changes in muscle length , not tension.
Thus, the correct answer is Ib sensory afferents , as they are responsible for transmitting information from Golgi tendon organs .
Which motor neuron type actually makes muscles contract ?
26 / 118
Category:
Neurosciences – Physiology
Which of the following types of neurons innervate extrafusal muscle fibers?
Extrafusal muscle fibers are the main contractile fibers of skeletal muscle and are directly responsible for force generation . These fibers are innervated by alpha motor neurons (α-motor neurons ), which originate in the anterior horn of the spinal cord and transmit signals via large, myelinated Aα fibers .
Function of Alpha Motor Neurons:
Directly stimulate extrafusal muscle fibers , causing muscle contraction .
Receive input from the corticospinal tract, brainstem motor pathways, and spinal reflex circuits .
Use acetylcholine (ACh) at neuromuscular junctions to trigger contraction.
Why not the other options?
A-delta neurons → Carry pain and temperature sensations, not motor control.
Gamma motor neurons → Innervate intrafusal muscle fibers (muscle spindles), which regulate muscle tone and proprioception , not extrafusal fibers.
C type nerve fibers → Unmyelinated fibers that transmit slow pain and temperature , not motor control.
A-beta neurons → Carry touch, vibration, and proprioception signals , not motor commands.
Thus, the correct answer is alpha motor neurons , as they directly control extrafusal muscle fiber contraction .
The patient understands speech but struggles to speak —which part of the brain controls speech production ?.. Can’t believe i would have to repeat hints, but hey… I am as washed up as this question bank 😮
27 / 118
Category:
NeuroSciences – Anatomy
A 45-year-old woman suffers from a stroke and has difficulty with speech. She can write and understand speech but is unable to produce any words. The area affected in this case is located in which of the following gyri of the cerebral cortex?
This 45-year-old woman presents with:
✅ Difficulty with speech production (expressive aphasia).
✅ Can write and understand speech → Comprehension is intact , meaning Wernicke’s area is not affected .
✅ Unable to produce words → Suggests a lesion in Broca’s area (Brodmann areas 44 & 45).
Location of Broca’s Area:
Found in the inferior frontal gyrus of the dominant hemisphere (usually the left).
Responsible for speech production and articulation .
Broca’s aphasia leads to non-fluent, effortful speech , but with intact comprehension .
Why not the other options?
Precentral gyrus → This is the primary motor cortex , which controls voluntary movement , not speech production.
Superior temporal gyrus → Contains Wernicke’s area , responsible for speech comprehension , not production.
Superior frontal gyrus → Involved in higher cognitive functions , not speech.
Middle frontal gyrus → Associated with executive functions and attention , not language processing.
Thus, the correct answer is inferior frontal gyrus , as it houses Broca’s area , which is responsible for speech production .
This cranial nerve is responsible for moving the eye and also controls pupil size and lens focusing —which one is it?
28 / 118
A person with damage to this area knows what they want to say but struggles to say it . What part of the brain controls the formation of speech?
29 / 118
Category:
NeuroSciences – Anatomy
A 42-year-old woman suffered a head injury in a road traffic accident. She was brought to the emergency room and underwent an MRI scan, which revealed a hemorrhage in the inferior frontal gyrus. Which of the following symptoms is the most likely manifestation of a lesion in this area of the cerebral cortex?
The inferior frontal gyrus of the dominant hemisphere (usually left) contains Broca’s area (Brodmann areas 44 & 45) , which is responsible for speech production .
A lesion in Broca’s area leads to Broca’s (expressive) aphasia , characterized by:
✅ Non-fluent speech → The patient struggles to form complete words and sentences.
✅ Comprehension intact → The patient understands speech but cannot express thoughts fluently .
✅ Frustration → The patient is aware of their speech difficulty.
✅ Right-sided hemiparesis (often mild) → Due to nearby motor cortex involvement.
Why not the other options?
Paralysis of pharynx and larynx → Associated with vagus nerve (CN X) lesions , not a frontal cortex lesion.
Paralysis of the upper limb → Would occur with a lesion in the precentral gyrus (primary motor cortex, upper limb region), not the inferior frontal gyrus .
Deviation of the eyes to the side of the lesion → Seen with frontal eye field (FEF) lesions (Brodmann area 8) , which is higher in the frontal lobe than Broca’s area.
Inability to grasp objects with her hands → Suggests apraxia , which is more associated with parietal lobe lesions , not Broca’s area.
Thus, the correct answer is “Inability to speak whole words,” as a hemorrhage in the inferior frontal gyrus likely affects Broca’s area , causing expressive aphasia .
A wild, flinging movement on one side of the body points to damage in a small but crucial part of the basal ganglia circuit —what structure normally suppresses such movements?
30 / 118
Category:
NeuroSciences – Anatomy
A 35-year-old man is brought to the emergency room after he suffers a head injury in a road traffic accident. It is observed that he has flailing movement in his left upper limb. Which of the following structures is the most likely site of lesion in this case?
The flailing movement of the left upper limb suggests hemiballismus , a condition caused by damage to the contralateral subthalamic nucleus .
Hemiballismus is characterized by sudden, involuntary, violent flinging movements of the proximal limbs , usually affecting one side of the body .
It results from damage to the subthalamic nucleus , which normally inhibits excessive movement via the indirect pathway of the basal ganglia .
A right-sided subthalamic lesion would cause left-sided hemiballismus .
Why not the other options?
Hypothalamus → Regulates autonomic and endocrine functions, not voluntary movement control.
Globus pallidus → Damage here can cause movement disorders , but not hemiballismus . The globus pallidus is more associated with Parkinsonism or dystonia .
Spinal cord → Lesions here cause weakness or sensory deficits , not flailing movements .
Midbrain → Damage here affects cranial nerves (e.g., oculomotor nerve in Weber’s syndrome) and dopaminergic pathways (e.g., Parkinson’s disease), but does not typically cause hemiballismus.
Thus, the subthalamus (subthalamic nucleus) is the correct answer, as its lesion leads to hemiballismus .
Before reaching the brain, visual information travels in a way that initially keeps things separate but later mixes signals from both sides. Where in this pathway does this crucial switch happen?
31 / 118
Think about a condition where neurons fire excessively due to reduced inhibition —leading to uncontrolled electrical activity in the brain.
32 / 118
Category:
Neurosciences – Biochemistry
Which of the following conditions is associated with low levels of gamma-aminobutyric acid (GABA)?
Gamma-aminobutyric acid (GABA) is the primary inhibitory neurotransmitter in the central nervous system. It reduces neuronal excitability by increasing chloride ion influx , leading to hyperpolarization of neurons .
Low GABA levels lead to increased neuronal excitability , which is a major factor in the pathophysiology of epilepsy .
Seizures occur due to uncontrolled, excessive neuronal firing , which is normally inhibited by GABAergic activity.
GABA and Epilepsy:
✅ Many anti-epileptic drugs (AEDs) work by enhancing GABA activity , such as:
Benzodiazepines (e.g., diazepam, lorazepam) → Increase GABA-A receptor activity.
Barbiturates (e.g., phenobarbital) → Prolong GABA-A receptor opening.
Vigabatrin → Inhibits GABA transaminase , preventing GABA breakdown.
Why not the other options?
Attention deficit hyperactivity disorder (ADHD) → More related to dopamine and norepinephrine dysregulation , not GABA deficiency.
Increased pain sensation → More associated with glutamate, substance P, and reduced endorphins , rather than low GABA.
Cerebral palsy → Primarily caused by brain injury or abnormal development , not directly due to low GABA.
Anxiety → While GABA is involved in anxiety regulation , it is not the primary cause; serotonin and amygdala dysfunction play a major role .
Thus, epilepsy is the correct answer , as it is strongly associated with low GABA levels and excessive neuronal excitability .
If a patient ignores one side of their body but has no paralysis , think about right parietal lobe damage
33 / 118
Category:
NeuroSciences – Anatomy
A 55-year-old male patient presents to the neurology clinic with the complaint of not being able to acknowledge the left side of his body. He does not wash, shave, or use the left side of his body. A lesion in which of the following areas of the cerebral cortex is the most likely cause for this deficit in the patient?
This patient is displaying hemispatial neglect syndrome , a condition where he fails to acknowledge or interact with one side of his body (left-sided neglect) . This typically results from a lesion in the right parietal lobe, specifically the somesthetic association area (Brodmann areas 5 & 7) , which is responsible for spatial awareness and integration of sensory information .
Key Features of Hemispatial Neglect:
✅ Neglects the left side of the body → Does not wash, shave, or use the left side.
✅ Usually caused by a lesion in the right parietal lobe (affecting the somesthetic association area ).
✅ Commonly associated with stroke affecting the middle cerebral artery (MCA) territory .
Why not the other options?
Primary somesthetic area (Postcentral gyrus, Brodmann areas 3, 1, 2) → Processes somatic sensation (touch, pain, temperature, proprioception) but does not cause neglect; damage would result in sensory loss, not spatial neglect .
Prefrontal gyrus → Likely referring to the prefrontal cortex , which is involved in executive function and decision-making , not spatial neglect.
Secondary somesthetic area → Involved in higher-order sensory processing , but not directly responsible for spatial awareness .
Precentral gyrus (Primary motor cortex, Brodmann area 4) → Controls voluntary movement , and damage here would cause motor deficits (hemiparesis), not neglect .
Thus, the correct answer is somesthetic association area , as this area in the right parietal lobe is responsible for spatial awareness and body schema integration .
Think of the fourth ventricle lying between the brainstem and cerebellum
34 / 118
Category:
NeuroSciences – Anatomy
A child is brought to the emergency room with complaints of severe vomiting and seizures. A radiologic scan revealed dilation of the fourth ventricle. Which of the following structures form(s) the floor of the fourth ventricle?
The fourth ventricle is a diamond-shaped cavity located between the brainstem and cerebellum . Its floor is formed by:
✅ Posterior surface of the pons
✅ Posterior surface of the medulla oblongata
This floor is also known as the rhomboid fossa , and it contains important structures such as:
Facial colliculus (formed by fibers of the facial nerve looping around the abducens nucleus).
Hypoglossal and vagal trigones (containing respective cranial nerve nuclei).
Obex (the inferior point of the fourth ventricle).
Why not the other options?
Inferior cerebellar peduncle → Forms part of the lateral wall of the fourth ventricle, not the floor.
Superior cerebellar peduncle → Forms part of the roof , not the floor.
Anterior surface of the cerebellum → The cerebellum forms the roof of the fourth ventricle, not the floor.
Superior and inferior medullary velum → These thin sheets of white matter form part of the roof , not the floor.
Thus, the correct answer is posterior surface of the pons and medulla , as these structures form the floor of the fourth ventricle .
If a patient has meningitis symptoms + TB history + MRI findings suggesting TB , What’s the treatment???—waiting can be fatal!
35 / 118
Category:
Neurosciences – Pathology
A 35-year-old housewife presents to the neurology clinic for evaluation of persistent headache and vomiting for one week with intermittent clouding of consciousness and confusion. She had a prior history of weight loss and fever for the past two months. Two years back, she was treated for pulmonary tuberculosis and became asymptomatic. On examination, signs of meningeal irritation are positive with bilateral papilledema. MRI brain contrast suggests central nervous system tuberculosis. Cerebrospinal fluid detailed report is awaited. What is the most appropriate next step in the management of this patient?
This 35-year-old woman presents with:
✅ Persistent headache, vomiting, clouding of consciousness → Suggestive of raised intracranial pressure (ICP) .
✅ History of weight loss, fever, and past tuberculosis → Indicates a chronic infectious etiology .
✅ Positive meningeal signs + bilateral papilledema → Suggests tuberculous meningitis (TBM) .
✅ MRI findings consistent with CNS tuberculosis → Strongly suggests TB meningitis or tuberculoma .
Since her clinical and imaging findings strongly suggest CNS tuberculosis , immediate initiation of anti-tuberculous therapy (ATT) is critical rather than waiting for CSF confirmation, which may take time.
Management of CNS Tuberculosis:
Start ATT immediately (RIPE regimen for 12 months):
Rifampin
Isoniazid (with pyridoxine to prevent neuropathy)
Pyrazinamide
Ethambutol
Adjunctive corticosteroids (Dexamethasone) for 6–8 weeks to reduce inflammation and prevent complications like hydrocephalus.
Monitor for complications → Hydrocephalus, infarcts, cranial nerve palsies.
Why not the other options?
Start anti-fungals → Fungal meningitis (e.g., Cryptococcus) occurs mostly in HIV/immunocompromised patients , and imaging would show basal meningitis with gelatinous material , not TB-specific findings.
Start steroids → Steroids are used as adjunctive therapy , but they should not be given alone without ATT.
Wait for CSF report → Delaying treatment can be fatal , as TBM has a high mortality rate if left untreated.
Get an EEG done → Not useful here, as the primary issue is infectious meningitis, not epilepsy .
Thus, the most appropriate next step is to start anti-tuberculous therapy (ATT) immediately .
“gold standard” – Think of the vice city cheatcode. ***** _ _ _ _ _ _
The Second One.
36 / 118
Category:
Neuroscience – Radiology
What is the major advantage of magnetic resonance imaging (MRI) over computed tomography (CT) scan?
The major advantage of MRI over CT is its superior soft tissue contrast and resolution , making it the best imaging modality for detecting soft tissue abnormalities , such as:
✅ Brain and spinal cord pathology (e.g., tumors, demyelination, ischemia)
✅ Ligament and tendon injuries
✅ Muscle and joint disorders
✅ Detailed imaging of internal organs without radiation
Why is MRI better for soft tissue imaging?
Uses strong magnetic fields and radio waves instead of ionizing radiation.
Can differentiate between gray and white matter in the brain better than CT.
Provides multi-planar imaging (sagittal, coronal, axial views) .
High sensitivity in detecting edema, tumors, and demyelinating diseases .
Why not the other options?
Quick modality → CT is much faster than MRI , making it the preferred choice in emergencies (e.g., trauma, stroke).
Does not require any prior preparation → MRI often requires screening for metal implants/pacemakers and may need contrast agents.
Superior in detection of calcified lesions → CT is superior for detecting calcifications and bone pathology , not MRI.
Use of ionizing radiation → MRI does not use ionizing radiation , whereas CT does.
Thus, the correct answer is “excellent soft tissue imaging with high contrast sensitivity,” making MRI the preferred choice for soft tissue evaluation.
It messes with vitamin K → Fix it with more vitamin K!
37 / 118
Category:
Neurosciences – Pharmacology
Vitamin K reverses the overdose of which of the following agents?
Vitamin K is the antidote for warfarin overdose , as warfarin is a vitamin K antagonist that inhibits the synthesis of clotting factors II, VII, IX, and X , as well as protein C and protein S in the liver.
Mechanism of Warfarin Reversal with Vitamin K:
Warfarin inhibits vitamin K epoxide reductase (VKORC1) , preventing the activation of vitamin K-dependent clotting factors.
Administering vitamin K (phytonadione) restores clotting factor synthesis , reversing warfarin’s anticoagulant effects.
For severe bleeding , fresh frozen plasma (FFP) or prothrombin complex concentrate (PCC) is also given for immediate clotting factor replacement .
Why not the other options?
Unfractionated heparin → Reversed by protamine sulfate , not vitamin K.
Clopidogrel → An antiplatelet (P2Y12 inhibitor) that is not reversed by vitamin K ; platelet transfusion may help in severe cases.
Eptifibatide & Tirofiban → GPIIb/IIIa inhibitors , affecting platelet aggregation; reversed by platelet transfusion , not vitamin K.
Thus, the correct answer is warfarin , as vitamin K specifically counteracts its anticoagulant effect.
A clear sensory level + UMN signs + autonomic dysfunction ?
38 / 118
Category:
NeuroSciences – Anatomy
A 20-year-old male presented to the neurology outpatient department with complaints of bilateral lower limb weakness and urinary incontinence for the last 4 days. On examination, he had decreased power in both lower limbs with increased tone, brisk reflex, and upgoing plantar. The sensory level was found at the xiphisternum. What is the most likely neurological localization in this case?
This 20-year-old male presents with:
✅ Bilateral lower limb weakness → Suggests a lesion affecting both sides.
✅ Increased muscle tone (spasticity) → Indicates upper motor neuron (UMN) involvement .
✅ Brisk reflexes & upgoing plantar (Babinski sign positive) → Classic UMN lesion signs .
✅ Urinary incontinence → Suggests involvement of autonomic pathways , commonly seen in spinal cord lesions .
✅ Sensory level at the xiphisternum (T7 level) → A clear sensory level strongly indicates a spinal cord lesion at or above this level.
Why is this a spinal cord lesion?
A well-defined sensory level is a hallmark of spinal cord pathology .
UMN signs below the lesion (spasticity, hyperreflexia, Babinski sign) indicate disruption of descending corticospinal tracts .
Autonomic dysfunction (urinary incontinence) suggests involvement of the descending autonomic fibers .
Why not the other options?
Peripheral nerves → Would cause lower motor neuron (LMN) signs (flaccidity, hyporeflexia, muscle wasting), which are absent here.
Brainstem → Would likely present with cranial nerve involvement (e.g., diplopia, facial weakness) , which this patient does not have.
Muscles → Myopathies cause proximal muscle weakness , usually without sensory loss, hyperreflexia, or incontinence .
Anterior horn cells → Would cause LMN signs (flaccid weakness, atrophy, fasciculations), not the spasticity and UMN signs seen here .
Thus, the correct answer is spinal cord , as the UMN signs, sensory level, and autonomic involvement strongly suggest a thoracic spinal cord lesion .
A post-infectious, ascending, flaccid paralysis with areflexia ? Think Guillain-Barré syndrome
39 / 118
Category:
Neuroscience – Radiology
A 25-year-old male is brought to the emergency department with complaints of all four limbs weakness for 3 days with a history of diarrhea 3 weeks before. On examination, he has flaccid areflexic quadriparesis and the sensations are intact with downgoing plantar. What should be the diagnostic investigation in this case?
This 25-year-old male presents with:
✅ Flaccid areflexic quadriparesis (weakness in all four limbs, absent reflexes).
✅ History of diarrhea 3 weeks prior → Suggestive of a preceding infection (e.g., Campylobacter jejuni) .
✅ Intact sensations with downgoing plantar reflex → Suggests a peripheral nervous system disorder rather than an upper motor neuron lesion .
These findings are highly suggestive of Guillain-Barré syndrome (GBS) , an acute inflammatory demyelinating polyneuropathy that follows an infection and leads to ascending paralysis .
Why is Nerve Conduction Study (NCS) the Best Diagnostic Test?
NCS shows features of demyelination , including:
✅ Prolonged F-wave latency
✅ Reduced conduction velocity
✅ Conduction block
✅ Prolonged distal latencies
It helps confirm the diagnosis of GBS and differentiate it from other neuromuscular disorders .
Why not the other options?
Serum electrolytes → Hypokalemia can cause weakness , but it usually presents with cramps and cardiac abnormalities , not a post-infectious, symmetrical, progressive paralysis.
Cerebrospinal fluid detailed report (CSF DR) → Would show albuminocytologic dissociation (high protein, normal WBC) in GBS, but it is not the first diagnostic test ; it is supportive rather than confirmatory.
MRI cervical spine → Useful for spinal cord pathology (e.g., transverse myelitis, cord compression) but unnecessary here as sensory function is intact .
CT brain → Not helpful since there are no cranial signs or upper motor neuron features .
Thus, nerve conduction studies (NCS) are the best diagnostic investigation for suspected Guillain-Barré syndrome .
A strong magnet + an electronic device inside the body = serious trouble .
40 / 118
Think about which nerve controls facial expressions and runs alongside the auditory nerve —
41 / 118
Fluent but meaningless speech, poor comprehension, and lack of awareness = Wernicke’s aphasia .
42 / 118
Category:
NeuroSciences – Anatomy
Destructive lesion of the Wernicke’s area results in which of the following?
A destructive lesion of Wernicke’s area leads to Wernicke’s (receptive) aphasia , which is characterized by impaired language comprehension but fluent speech that lacks meaning (“word salad”) .
Wernicke’s area is located in the posterior superior temporal gyrus of the dominant hemisphere (usually left hemisphere, Brodmann area 22) .
It is responsible for processing and understanding spoken and written language .
Clinical Features of Wernicke’s Aphasia:
✅ Fluent but nonsensical speech → Patients speak in long, rambling sentences with incorrect or made-up words (neologisms) .
✅ Poor comprehension → They cannot understand spoken or written language .
✅ Lack of awareness → Patients are often unaware of their speech deficits .
✅ Intact motor function → Unlike Broca’s aphasia , speech production is fluent.
Why not the other options?
Global aphasia → Affects both Wernicke’s and Broca’s areas , causing loss of both speech production and comprehension .
Astereognosis → Inability to recognize objects by touch, due to parietal lobe damage , not Wernicke’s area.
Agraphia → Inability to write, commonly associated with angular gyrus lesions , not Wernicke’s aphasia.
Expressive aphasia (Broca’s aphasia) → Impaired speech production , but comprehension is intact , due to Broca’s area lesion (inferior frontal gyrus).
Thus, the correct answer is receptive aphasia , as Wernicke’s area is essential for language comprehension .
This theory suggests that bodily reactions dictate emotions —you don’t cry because you’re sad; you feel sad because you cry .
43 / 118
Category:
Neurosciences – Community Medicine + Behavioural Sciences
Ali saw a snake and he started trembling. The statement that Ali is afraid because he trembled is better explained by which of the following theories?
The James-Lange theory of emotion states that physiological arousal occurs first , followed by the experience of emotion . According to this theory, Ali is afraid because he trembled , meaning his body reacts first, and then he interprets the emotion based on his physiological response.
Example :
Event : Ali sees a snake 🐍.
Physiological response : His body trembles, heart rate increases, and adrenaline surges.
Emotion : Ali feels fear because he perceives his bodily reaction.
Why not the other options?
Cannon-Bard theory → States that physiological arousal and emotion happen simultaneously , not sequentially. (Ali would feel fear at the same time his body trembles ).
Darwin’s theory → Focuses on evolutionary aspects of emotions (e.g., survival advantages), not the sequence of physiological responses and emotions.
Two-factor theory (Schachter-Singer theory) → Suggests that physiological arousal is interpreted based on context before experiencing an emotion . (Ali would need to consciously label the trembling as fear ).
Appraisal theory → States that cognitive evaluation of the situation determines the emotional response , rather than bodily reactions leading to emotions.
Thus, the James-Lange theory is the correct answer, as it explains that Ali feels fear because his body trembles first .
Great ideas don’t just appear instantly —they require gathering knowledge, subconscious thinking, breakthrough moments, evaluation, and application .
44 / 118
Category:
Neurosciences – Community Medicine + Behavioural Sciences
As a medical professional, you have to come up with ideas for research that are original, unique, new, and relevant to your field of practice. This involves creative thinking. Which of the following will you follow?
Accommodation, centration, egocentrism, incubation, revision
Preparation, accommodation, assimilation, centration, egocentrism
Preparation, incubation, illumination, evaluation, implementation
Incubation, preparation, revision, accommodation, centration
Assimilation, centration, egocentrism, incubation, revision
The process of creative thinking follows a well-established model in psychology and problem-solving, often referred to as Wallas’ Stages of Creativity , which includes the following steps:
Preparation → Gathering knowledge, researching existing literature, and identifying the problem.
Incubation → Subconscious processing, where ideas form without active effort.
Illumination → The “Aha!” moment when a creative idea or solution emerges.
Evaluation → Critically assessing the idea for feasibility and relevance.
Implementation → Applying the idea to research, experimentation, or practice.
Why not the other options?
Preparation, accommodation, assimilation, centration, egocentrism → Includes Piagetian cognitive development terms , not creativity stages.
Incubation, preparation, revision, accommodation, centration → Does not follow the structured flow of creative thinking.
Assimilation, centration, egocentrism, incubation, revision → Again, Piagetian cognitive development terms mixed with irrelevant steps.
Accommodation, centration, egocentrism, incubation, revision → Focuses more on child cognitive development , not creativity.
Thus, the correct answer is preparation, incubation, illumination, evaluation, implementation , as it follows the natural process of creative thinking and research development .
Think of vasculitis cutting off blood supply to different nerves at different sites , leading to patchy, asymmetric nerve involvement .
45 / 118
Category:
Neurosciences – Pathology
Peripheral nerves are often damaged in many different forms of different vasculitis including polyarteritis nodosa (PAN), Churg-Strauss syndrome, and Wegener’s granulomatosis. Which of the following is the most common clinical picture in these disorders?
Vasculitic disorders such as polyarteritis nodosa (PAN) , eosinophilic granulomatosis with polyangiitis (Churg-Strauss syndrome) , and granulomatosis with polyangiitis (Wegener’s) can cause peripheral nerve damage due to inflammation and ischemia of the vasa nervorum — the small blood vessels supplying peripheral nerves.
🧠 Why Mononeuritis Multiplex is the Most Common:
Mononeuritis multiplex refers to asymmetric, asynchronous involvement of multiple individual peripheral nerves.
This is characteristic of vasculitic neuropathy because:
Blood vessels supplying nerves are focally affected .
This leads to patchy ischemia , damaging one nerve at a time.
Over time, different nerves in different areas are involved in a scattered pattern.
For example:
A patient may first lose function in the right radial nerve (wrist drop), then later develop issues with the left peroneal nerve (foot drop).
This asymmetry and multiplicity are hallmarks of mononeuritis multiplex .
❌ Why the Other Options Are Incorrect:
1. Loss of sensation in limbs:
2. Autonomic neuropathy:
3. Combination neuropathy:
4. Polyneuropathy:
Refers to symmetric, distal nerve involvement , like in diabetes or toxic/metabolic causes .
Vasculitis causes focal ischemia , making symmetric involvement less likely early in the disease.
You can’t change your genes, gender, or age , but you can change your weight and lifestyle .
46 / 118
Category:
Neurosciences – Community Medicine + Behavioural Sciences
Which of the following is a modifiable risk factor for stroke?
Among the options listed, obesity is the only modifiable risk factor for stroke . Modifiable risk factors are those that can be changed or controlled through lifestyle modifications or medical interventions.
Why is obesity a modifiable risk factor?
Increases the risk of hypertension, diabetes, and hyperlipidemia , which are all major contributors to stroke.
Promotes atherosclerosis , leading to ischemic stroke.
Can be managed through lifestyle changes , such as diet, exercise, and medical therapy .
Why not the other options?
Genetics → A non-modifiable factor, as inherited predispositions cannot be changed.
Gender → Men have a higher stroke risk , but this is not modifiable .
Family history → If close relatives have had strokes, the individual is at increased risk, but this cannot be altered .
Age → Older age is a major risk factor for stroke , but it cannot be changed .
Thus, the correct answer is obesity , as it is a modifiable risk factor that can be improved through lifestyle and medical interventions.
A teenager with chronic headaches and seizures , and a brain mass with past and recent bleeds —think of a hidden ticking time bomb in the brain’s blood supply .
47 / 118
Category:
Neurosciences – Pathology
A 15-year-old girl presented to the outpatient department with complaints of headaches for the past 8 months. Over a 2-week period, she has a generalized seizure. On physical examination, there is no papilledema or movement disorder. Computed tomography (CT) scan of the head shows a 2 cm mass in the right frontal lobe and shows evidence of recent and remote hemorrhage. What is the most likely diagnosis?
This 15-year-old girl presents with chronic headaches for 8 months , followed by a generalized seizure , and a frontal lobe mass with evidence of both recent and remote hemorrhage on CT scan. This combination is highly suggestive of an arteriovenous malformation (AVM) .
Key Features of AVM:
Congenital vascular malformation with tangled arteries and veins .
Prone to spontaneous bleeding , leading to:
Headaches (due to chronic microbleeds).
Seizures (due to cortical irritation from bleeding).
Most common in the supratentorial region (frontal or parietal lobes) .
CT/MRI findings:
Irregular mass-like lesion .
Evidence of past and recent hemorrhage .
Why not the other options?
Organizing abscess → Would show a ring-enhancing lesion , usually with fever and infection signs , which are absent here.
Prior head trauma → Could cause a chronic subdural hematoma , but would not form a discrete vascular mass .
Ruptured saccular aneurysm → Typically causes sudden-onset thunderclap headache and subarachnoid hemorrhage , not a slow-growing frontal mass with seizures .
Angiosarcoma → A rare malignant vascular tumor that presents with rapid growth and systemic symptoms , unlike this chronic presentation.
Thus, the most likely diagnosis is arteriovenous malformation (AVM) , as it explains the gradual progression of headaches, seizure onset, and the CT findings of hemorrhage in a young patient .
When a patient is completely unresponsive, someone has to step in and take control of the decisions —but who gets that power, and on what basis?
48 / 118
Category:
Neurosciences – Community Medicine + Behavioural Sciences
An 85-year-old male is in a coma in a hospital. His daughter makes all the decisions regarding his treatment on his behalf. Which of the following options best describes the type of consent in this scenario?
Since the 85-year-old male is in a coma , he is unable to provide consent for his medical decisions. In this case, his daughter is making medical decisions on his behalf , which is classified as surrogate consent .
Surrogate Decision-Making:
A surrogate (proxy) decision-maker is typically a legally authorized family member (such as a spouse, adult child, or designated healthcare proxy) who makes medical decisions when a patient is incapable of doing so .
The hierarchy of surrogates varies by jurisdiction but generally follows: spouse > adult children > parents > siblings .
Surrogates should make decisions based on the patient’s previously expressed wishes or best interests .
Why not the other options?
Expressed consent → Occurs when a patient directly agrees to treatment verbally or in writing , which is not possible in this case.
Informed consent → Requires a competent patient to understand risks, benefits, and alternatives before agreeing to treatment, which the patient cannot do due to being in a coma.
Opt-out consent → Used in specific cases like organ donation , where consent is assumed unless the individual explicitly refuses. This scenario does not involve opt-out consent.
Implied consent → Assumed in emergency situations when the patient is incapacitated , and immediate treatment is necessary to prevent harm. Since the patient’s daughter is actively making decisions , this is a case of surrogate consent , not implied consent.
Thus, the correct answer is surrogate consent , as the daughter is legally making medical decisions for her incapacitated father.
CN III compression → “Blown pupil” (mydriasis) + impaired eye movement .
49 / 118
Category:
Neurosciences – Pathology
A 39-year-old woman has had a severe headache for 3 days. On physical examination, she is afebrile and normotensive. Fundoscopic examination shows papilledema on the right. One day later, she has right pupillary dilation and impaired ocular movement. Which of the following lesions best explains these findings?
This patient’s progressive headache, papilledema, and later onset of right-sided pupillary dilation with impaired ocular movement strongly suggest increased intracranial pressure (ICP) leading to transtentorial (uncal) herniation .
Mechanism of Uncal Herniation:
The uncus (medial temporal lobe) is displaced downward through the tentorial notch , compressing adjacent structures:
✅ Oculomotor nerve (CN III) compression → Results in pupillary dilation (mydriasis) and ophthalmoplegia due to loss of parasympathetic innervation.
✅ Compression of the posterior cerebral artery (PCA) → Can lead to occipital lobe infarction and contralateral homonymous hemianopia .
✅ Midbrain displacement → May cause Duret hemorrhages in the brainstem.
Why not the other options?
Ruptured middle cerebral berry aneurysm → More likely to present with sudden-onset severe headache (“thunderclap headache”) , followed by subarachnoid hemorrhage signs like nuchal rigidity , but it does not typically cause progressive papilledema and CN III palsy .
Chronic subdural hematoma → Causes gradual symptoms (weeks to months) , often with altered mental status rather than isolated ICP signs and CN III involvement .
Frontal lobe abscess → Would likely present with fever, leukocytosis, and localized neurological deficits , not progressive uncal herniation signs .
Hydrocephalus ex vacuo → Seen in cortical atrophy (e.g., Alzheimer’s, Huntington’s disease) with ventricular enlargement , but not associated with increased ICP, papilledema, or herniation .
Thus, the correct answer is transtentorial medial temporal (uncal) herniation , as it best explains increased ICP, papilledema, and CN III palsy .
Think of opioid overdose symptoms
50 / 118
Category:
Neurosciences – Pharmacology
Which of the following statements is correct regarding the mu receptor stimulation?
The mu (μ) opioid receptor is the primary receptor responsible for the analgesic and side effects of opioids , including respiratory depression , which is the most serious adverse effect of opioid use.
Effects of Mu (μ) Receptor Stimulation:
✅ Respiratory depression → Inhibition of the brainstem respiratory centers reduces the drive to breathe, making this a life-threatening side effect of opioid overdose.
Other effects of μ receptor activation include:
Analgesia (supraspinal and spinal)
Euphoria (reinforcing effect leading to addiction)
Sedation
Miosis (pupil constriction)
Bradycardia and hypotension
Constipation (due to decreased GI motility)
Why not the other options?
Analgesia (spinal) → While opioids do produce spinal analgesia, mu receptors primarily mediate supraspinal analgesia (via the periaqueductal gray and rostral ventromedial medulla). Spinal analgesia is mainly mediated by kappa (κ) receptors .
Sedation → A side effect of μ receptor activation, but respiratory depression is a more direct and critical effect .
Miosis → Caused by mu receptor activation in the Edinger-Westphal nucleus , but not the most defining feature of mu receptor stimulation.
Hallucinations → Associated more with kappa (κ) receptor stimulation , not mu receptors.
Thus, the correct answer is respiratory depression , as it is one of the most dangerous effects of mu receptor stimulation
Doctors must communicate in a way which avoids misleading patients into unnecessary treatments .
51 / 118
Category:
Neurosciences – Community Medicine + Behavioural Sciences
A surgeon working in a private rehabilitation facility encounters a patient with a certain tumor that responds only to radiotherapy and surgery. The surgeon is enthusiastic to perform the surgery and proceeds to convince the patient to perform the surgery despite the lack of evidence of surgery being the better option. Which of the following principles of ethical communication has the surgeon breached in the above scenario?
The surgeon breached the principle of honesty and truthfulness by persuading the patient to undergo surgery despite a lack of evidence that it is the best treatment option . Ethical medical communication requires providing complete, unbiased, and truthful information so that the patient can make an informed decision .
Medical ethics emphasize transparency , and a physician must not mislead or withhold critical information to influence a patient’s decision.
Why not the other options?
Beneficence → Refers to acting in the best interest of the patient . While the surgeon may believe surgery is beneficial, they are not prioritizing the patient’s best interest due to lack of evidence .
Equity and justice → Concern fairness and equal access to care , but this scenario is more about misleading the patient rather than injustice.
Active listening → Involves listening to the patient’s concerns , but the ethical violation here is about providing false or misleading information, not a failure to listen .
Empathy → Involves understanding and sharing the patient’s emotions , but the surgeon’s issue here is lack of honesty, not lack of empathy .
Thus, the correct answer is honesty and truthfulness , as the surgeon has misled the patient by promoting an unproven surgical intervention .
Think about the type of prevention that focuses on stopping a disease from occurring in the first place. This strategy involves lifestyle changes and healthy habits to reduce the risk of developing conditions like hypertension and stroke. Which level of prevention aligns with this approach?
52 / 118
a synaptic messenger with specific receptors and pharmacological modulation (agonists/antagonists).
53 / 118
Category:
Neurosciences – Physiology
A compound that is synthesized in a neuron, has a receptor in vicinity, and has agonists and antagonists defines which of the following terms?
A neurotransmitter is a chemical compound synthesized in a neuron , released at the synapse, and acts on specific receptors in the vicinity (i.e., the postsynaptic neuron or effector cell). It also has agonists and antagonists that can modulate its function.
Criteria for a Neurotransmitter:
Synthesized in a neuron .
Stored in vesicles and released upon nerve stimulation .
Binds to a specific receptor on the target cell .
Has agonists (activators) and antagonists (blockers) .
Effect is terminated by enzymatic degradation, reuptake, or diffusion .
Why not the other options?
Neurohormone → Released by neurons but acts on distant targets via the bloodstream (e.g., oxytocin, vasopressin).
Neuromediator → A vague term, sometimes used interchangeably with neurotransmitters, but does not specifically imply the presence of receptors, agonists, and antagonists.
Neuropeptide → A subclass of neurotransmitters that are peptides (e.g., substance P, endorphins) but the question refers to the general definition of a neurotransmitter.
Neurochemical mediator → A broader term that includes neurotransmitters, neuropeptides, and neuromodulators but is not a specific classification.
Thus, the correct answer is neurotransmitter , as it best fits the definition provided.
preventing the use of long-chain fatty acids as an energy source.
54 / 118
Category:
Neurosciences – Biochemistry
What is the reason that brain tissue cannot utilize fatty acids for energy?
The brain cannot utilize fatty acids for energy because long-chain fatty acids cannot cross the blood-brain barrier (BBB) efficiently. Instead, the brain primarily relies on glucose as its main energy source and can use ketone bodies during prolonged fasting or starvation.
Key Reasons Why Fatty Acids Are Not Used for Energy in the Brain:
Blood-Brain Barrier Restriction → The BBB prevents long-chain fatty acids from entering the brain , making them unavailable for mitochondrial beta-oxidation in neurons.
Lack of Beta-Oxidation Enzymes → Even if fatty acids enter, neurons have low levels of beta-oxidation enzymes , making it difficult to metabolize them.
Primary Energy Sources :
Glucose → The brain’s preferred energy source under normal conditions.
Ketone Bodies (e.g., β-hydroxybutyrate, acetoacetate) → Used during prolonged fasting or starvation when glucose levels are low.
Why not the other options?
Enzymes of beta-oxidation are present in the brain → Incorrect. Neurons lack significant beta-oxidation activity , meaning even if fatty acids entered, they wouldn’t be efficiently used for energy.
They are not found in brain tissue → Incorrect. Fatty acids are present in the brain but mainly used for membrane synthesis, not energy production .
Fatty acids are attached with albumin in blood → While true, this does not prevent other tissues (like muscle or liver) from utilizing them; the key issue is BBB restriction .
They are utilized in ganglioside formation → True, but this does not explain why they cannot be used for energy.
Thus, the correct answer is “They cannot pass through the blood-brain barrier” , as this is the primary reason why the brain does not utilize fatty acids for energy.
Think about the enzyme that creates acetylcholine from choline and acetyl-CoA
55 / 118
Think of the biological clock (SCN) that controls the light-dark cycle and melatonin release .
56 / 118
Category:
NeuroSciences – Anatomy
Secretion of melatonin by the pineal gland is regulated by which one of the following parts of the diencephalon?
While the pineal gland (located in the epithalamus ) secretes melatonin , its secretion is regulated by the hypothalamus , specifically by the suprachiasmatic nucleus (SCN) .
The suprachiasmatic nucleus (SCN) of the hypothalamus acts as the body’s master clock for regulating circadian rhythms.
It receives light signals from the retina via the retinohypothalamic tract , which then regulates pineal gland activity through a pathway involving:
SCN (hypothalamus) → Paraventricular nucleus (PVN) → Spinal cord (sympathetic neurons)
Sympathetic fibers from the superior cervical ganglion stimulate the pineal gland
Pineal gland secretes melatonin in response to darkness (inhibited by light).
Why not the other options?
Globus pallidus → Part of the basal ganglia , involved in motor control, not melatonin regulation.
Epithalamus → Houses the pineal gland , which secretes melatonin, but the regulatory control comes from the hypothalamus (SCN) .
Subthalamus → Involved in motor function , not circadian rhythm or melatonin regulation.
Metathalamus → Includes the medial and lateral geniculate nuclei , which process auditory and visual information , unrelated to melatonin secretion.
Thus, the hypothalamus (specifically the SCN) is the primary regulator of melatonin secretion from the pineal gland.
57 / 118
Category:
NeuroSciences – Anatomy
Which of the following sensations are relayed at the ventral posterolateral nuclei of the thalamus before reaching the post-central gyrus?
The ventral posterolateral (VPL) nucleus of the thalamus relays somatic sensory information from the body (trunk and limbs) to the primary somatosensory cortex (postcentral gyrus, Brodmann areas 3, 1, 2) .
Pathways that Synapse at the VPL Nucleus:
Dorsal column-medial lemniscus (DCML) pathway → Fine touch, vibration, and proprioception from the trunk and limbs .
Spinothalamic tract → Pain, temperature, and crude touch from the trunk and limbs .
Why not the other options?
Somatic afferents from head and arms → Somatic afferents from the head (face and oral structures) are relayed by the ventral posteromedial (VPM) nucleus , not VPL.
Visceral afferents from the gut (duplicated option) → Visceral sensory information is relayed via the intralaminar and dorsal thalamic nuclei , not the VPL.
Somatic afferents from head and limbs → The head (face) is relayed via VPM , while the limbs go to VPL . Since the question asks for a single thalamic nucleus, the correct answer should focus on VPL , which relays trunk and limb sensory input .
Thus, the ventral posterolateral (VPL) nucleus of the thalamus relays somatic afferents from the trunk and limbs before sending them to the postcentral gyrus (somatosensory cortex) .
Think about how nerve impulse transmission is stopped..
so think about this.. What prevents action potential propagation.
58 / 118
Category:
Neurosciences – Physiology
During suturing of a wound, a patient receives local anesthesia around the wound area. After this, he did not feel any needle pain during suturing. Which of the following option could be the reason for this?
Local anesthetics (e.g., lidocaine, bupivacaine, procaine) work by blocking voltage-gated Na⁺ channels in peripheral nerve fibers , preventing the generation and propagation of action potentials. This inhibits pain transmission from the wound site to the central nervous system.
Mechanism of Action of Local Anesthetics:
Local anesthetic diffuses into the nerve membrane .
Binds to intracellular voltage-gated Na⁺ channels .
Prevents Na⁺ influx , blocking action potential initiation.
Sensory transmission is inhibited , leading to loss of pain sensation in the affected area.
Why not the other options?
Opening of ligand-gated Na⁺ channels → Would lead to nerve excitation , increasing pain sensation instead of blocking it.
Blockage of ligand-gated K⁺ channels → Would affect neurotransmission but is not the primary mechanism of local anesthetics.
Early closure of voltage-gated Na⁺ channels → While similar, local anesthetics prevent opening , rather than causing early closure.
Blockage of voltage-gated K⁺ channels → Would prolong action potentials , affecting repolarization, but not prevent pain transmission like Na⁺ channel blockade does.
Thus, blocking voltage-gated Na⁺ channels is the primary mechanism by which local anesthetics prevent pain sensation .
Think about which pathway transmits crude touch rather than fine, discriminatory touch
59 / 118
Category:
NeuroSciences – Anatomy
During an examination of a young boy, the child feels very ticklish so it becomes difficult for the doctor to examine him. Which of the following tract carries the above sensation?
The anterior (ventral) spinothalamic tract is responsible for transmitting crude touch and pressure sensations , including tickling and light touch .
Crude touch refers to light, poorly localized touch , such as a feather brushing against the skin or a ticklish sensation.
Fine touch , on the other hand, which is well-localized and discriminatory (e.g., recognizing textures), is carried by the dorsal column-medial lemniscus (DCML) system .
Why not the other options?
Corticospinal tract → Controls voluntary motor function , not sensation.
Lateral spinothalamic tract → Transmits pain and temperature , not crude touch or tickling.
Dorsal column-medial lemniscus system → Transmits fine touch, proprioception, and vibration , not crude touch or tickling.
Spinocerebellar tract → Transmits unconscious proprioception for coordination, not touch sensation.
Thus, the anterior spinothalamic tract is the correct answer, as it carries the ticklish sensation experienced by the child.
Think about the ion that enters through NMDA receptors and is required for synaptic strengthening
60 / 118
Category:
Neurosciences – Physiology
Long-term potentiation is identified in hippocampal mossy fibers by an increase in which of the following ions in the postsynaptic area?
Long-term potentiation (LTP) is a fundamental mechanism of synaptic plasticity , critical for learning and memory , and occurs in the hippocampus , particularly at mossy fiber synapses (which connect the dentate gyrus to the CA3 region).
LTP is primarily driven by an increase in calcium (Ca²⁺) influx in the postsynaptic neuron .
Mechanism of Calcium Involvement in LTP:
Glutamate release from presynaptic neurons activates AMPA receptors , leading to sodium (Na⁺) influx and depolarization.
Depolarization removes the Mg²⁺ block from NMDA receptors , allowing Ca²⁺ influx into the postsynaptic neuron.
Increased intracellular Ca²⁺ activates signaling pathways (CaMKII, PKC ), leading to:
Enhanced AMPA receptor activity (increasing synaptic strength).
Increased neurotransmitter release from presynaptic terminals .
Gene expression changes that strengthen synaptic connections .
Why not the other options?
Potassium (K⁺) → Important for repolarization, but does not mediate LTP.
Magnesium (Mg²⁺) → Blocks NMDA receptors at resting membrane potential but is removed upon depolarization; it does not increase in the postsynaptic area.
Sodium (Na⁺) → Enters through AMPA receptors and contributes to depolarization but does not directly trigger LTP.
Phosphorus (P) → Involved in phosphorylation of proteins but not a key ion in LTP.
Thus, calcium (Ca²⁺) influx in the postsynaptic neuron is the essential trigger for long-term potentiation in hippocampal mossy fibers, making it the correct answer.
The sympathetic system prepares the body for “fight or flight”
61 / 118
Category:
Neurosciences – Physiology
Which of the following functions is the result of an increased activity of the sympathetic nervous system?
The sympathetic nervous system (SNS) is responsible for the “fight or flight” response , which prepares the body for emergencies by increasing alertness, heart rate, and energy availability while inhibiting non-essential functions like digestion and reproduction.
Effects of Sympathetic Activation:
✅ Bronchiolar dilation → The SNS stimulates β₂-adrenergic receptors , causing relaxation of bronchiolar smooth muscle , which increases airflow to the lungs for better oxygenation during stress or exercise.
Why not the other options?
Pupillary constriction → Controlled by the parasympathetic system via the oculomotor nerve (CN III, Edinger-Westphal nucleus) .
Gallbladder emptying → Controlled by the parasympathetic system , which promotes digestion by stimulating bile release.
Accommodation for near vision → A parasympathetic function controlled by the ciliary muscles .
Penile erection → Mediated by the parasympathetic system (Point = Parasympathetic , Shoot = Sympathetic for ejaculation).
Thus, bronchiolar dilation is the correct answer, as it is a classic sympathetic nervous system response to increase oxygen intake.
Think of the brain structure essential for memory formation —damage leads to anterograde amnesia .
62 / 118
Category:
NeuroSciences – Anatomy
During the removal of a brain tumor in a patient, a part of the brain was removed. Upon successful awakening, the patient is unable to form new memories. However, he can recall most previously learned memories satisfactorily. Which of the brain area may have been affected during the operation?
The hippocampus is the key brain structure responsible for forming new declarative (explicit) memories , including both episodic (personal experiences) and semantic (facts and knowledge) memories.
Damage to the hippocampus results in anterograde amnesia , meaning the patient cannot form new memories after the event but retains previously stored memories.
This aligns with the patient’s symptoms: inability to form new memories but intact recall of old ones .
Why not the other options?
Anterior hypothalamic nucleus → Involved in temperature regulation , not memory.
Suprachiasmatic nucleus → Regulates circadian rhythms (sleep-wake cycle) , not memory formation.
Epithalamus → Includes the pineal gland , which regulates melatonin and circadian rhythms, not memory.
Arcuate nucleus → Involved in hormone regulation and appetite control , not memory.
Thus, the hippocampus is the correct answer, as it plays a central role in consolidating new memories .
Think about fluent aphasia —speech is fluent but nonsensical , and comprehension is impaired .
63 / 118
Category:
Neurosciences – Physiology
A patient presents with the inability to interpret thoughts, although he is capable of expressing individual words or even short phrases. Which area of the brain is most likely to be affected?
The inability to interpret thoughts while still being able to express individual words or short phrases suggests a deficit in language comprehension , characteristic of Wernicke’s aphasia .
Wernicke’s area (Brodmann area 22) is located in the posterior part of the superior temporal gyrus in the dominant hemisphere (usually the left).
It is responsible for language comprehension and interpretation of speech.
Damage to Wernicke’s area leads to fluent aphasia (Wernicke’s aphasia), where speech is fluent but lacks meaning (word salad ) and comprehension is impaired.
Why not the other options?
Angular gyrus → Important for reading and writing ; damage here leads to alexia (inability to read) and agraphia (inability to write) , but not the described language deficit.
Broca’s area → Controls speech production ; damage leads to Broca’s aphasia (non-fluent speech with intact comprehension) , which is not the case here.
Arcuate fasciculus → Connects Wernicke’s and Broca’s areas ; damage causes conduction aphasia , characterized by difficulty repeating words , but comprehension remains intact.
Exner’s area → Involved in writing coordination , not speech comprehension.
Thus, the correct answer is Wernicke’s area , as it is crucial for interpreting language and forming meaningful speech .
Think about how the brain filters out background noise or repetitive stimuli to focus on important information
64 / 118
lack the protective myelin sheath, allowing anesthetic agents to more easily block sodium channels and inhibit nerve conduction.
65 / 118
Category:
Neurosciences – Physiology
Which of the following nerve fiber types is most sensitive to anesthetic agents?
C fibers are the most sensitive to anesthetic agents among the options listed.
C fibers : These are small, unmyelinated nerve fibers that transmit slow, dull, or aching pain signals. Due to their small diameter and lack of myelination, they are highly susceptible to local anesthetics. The anesthetic agents can easily block the sodium channels in these fibers, preventing the propagation of action potentials.
A delta fibers : These are small, myelinated fibers that transmit sharp, fast pain signals. They are less sensitive to anesthetics compared to C fibers because their myelination provides some protection against the blocking effects of anesthetics.
A gamma fibers : These are involved in muscle spindle function and are part of the motor system. They are myelinated and larger in diameter, making them less sensitive to anesthetics than C fibers.
A alpha fibers : These are large, myelinated fibers responsible for motor function and proprioception. Due to their large diameter and myelination, they are the least sensitive to anesthetic agents among the options listed.
B fibers : These are small, myelinated fibers that are part of the autonomic nervous system. They are more sensitive than A fibers but less sensitive than C fibers.
In summary, the order of sensitivity to anesthetic agents from most to least sensitive is: C fibers > B fibers > A delta fibers > A gamma fibers > A alpha fibers .
Think about the receptor type found at all autonomic ganglia , whether sympathetic or parasympathetic
66 / 118
Category:
Neurosciences – Physiology
Which type of cholinergic receptor is found at synapses between preganglionic and postganglionic neurons of the sympathetic system?
At synapses between preganglionic and postganglionic neurons of the sympathetic system , the neurotransmitter acetylcholine (ACh) is released by preganglionic fibers and binds to nicotinic cholinergic receptors (Nn subtype) on postganglionic neurons .
These nicotinic (Nn) receptors are ligand-gated ion channels that mediate fast synaptic transmission, allowing sodium (Na⁺) and potassium (K⁺) influx, leading to neuronal depolarization.
Why not the other options?
Muscarinic → Found on effector organs (e.g., heart, glands, smooth muscles) in the parasympathetic system and in sweat glands (sympathetic exception) but not at ganglionic synapses .
Alpha (α) → Adrenergic receptors (α1 and α2 ) are found on target organs and respond to norepinephrine (NE), not ACh .
Beta 2 (β2) → Adrenergic receptor found in smooth muscle (bronchi, blood vessels) and not at ganglionic synapses .
Beta 1 (β1) → Adrenergic receptor found in the heart and kidneys , involved in increasing heart rate and renin secretion.
Thus, nicotinic (Nn) receptors are the correct answer as they mediate synaptic transmission at autonomic ganglia in both sympathetic and parasympathetic systems .
Sweat glands are a unique exception in the sympathetic system
67 / 118
Think of the midbrain nucleus that influences voluntary movements, particularly of the upper limb flexors —this is the red nucleus , which gives rise to the rubrospinal tract .
68 / 118
Think about unconscious, rule-based learning that occurs effortlessly in childhood..
69 / 118
Category:
Neurosciences – Community Medicine + Behavioural Sciences
Young children can immediately repeat short sentences spoken by their parents and siblings, and also produce new sentences that follow the rules of their native language. The ability to produce new, rule-governed sentences is thought to involve which kind of learning?
The ability of young children to unconsciously absorb and apply language rules without formal instruction suggests implicit learning .
Implicit learning occurs without conscious effort and involves acquiring grammatical rules, sentence structures, and phonetics simply by exposure to language .
This type of learning relies on procedural memory and is heavily associated with the basal ganglia and Broca’s area .
Young children do not explicitly analyze grammar ; instead, they internalize language structures automatically through repeated exposure.
Why not the other options?
Short-term learning → Refers to temporarily holding information, such as repeating a sentence, but does not explain rule-based sentence generation.
Explicit learning → Requires conscious effort (e.g., studying grammar rules), but young children learn language without direct instruction .
Working memory → Holds information for a short time for processing (e.g., temporarily remembering a sentence) but does not explain how children form new sentences following rules .
Long-term learning → Refers to storing knowledge over extended periods, but implicit learning better describes how children acquire language rules unconsciously .
Thus, the correct answer is implicit learning , as it governs the natural acquisition of language rules in early childhood .
Think about the brain region involved in decision-making, attention, and short-term cognitive tasks
70 / 118
Category:
Neurosciences – Physiology
Maintaining working memory contents for a dozen seconds requires which of the following?
Working memory refers to the temporary storage and manipulation of information for short periods (seconds to minutes) and is primarily controlled by the prefrontal cortex .
The dorsolateral prefrontal cortex (DLPFC) plays a critical role in holding and processing information over short durations, such as when solving problems, following conversations, or remembering a phone number for a few seconds.
Why not the other options?
Amygdala → Involved in emotion processing and memory modulation , but not working memory.
Cerebellum → Plays a role in motor coordination and procedural learning , not short-term memory storage.
Medial temporal lobe → Important for long-term memory consolidation , especially in the hippocampus , but not working memory.
Hippocampus → Essential for long-term memory formation , particularly declarative memory, but not for maintaining short-term working memory .
Thus, the prefrontal cortex , specifically the dorsolateral prefrontal cortex (DLPFC) , is the correct answer as it is responsible for maintaining working memory contents for a dozen seconds .
Think of REM sleep’s role —without it, cognitive function declines .
71 / 118
Category:
Neurosciences – Physiology
Prolonged rapid eye movement (REM) sleep deprivation can result in which of the following?
Rapid eye movement (REM) sleep plays a crucial role in memory consolidation , particularly for procedural and emotional memory . Prolonged REM sleep deprivation can lead to:
Cognitive deficits , including impaired memory and learning difficulties .
Increased emotional reactivity and irritability .
REM rebound (when REM sleep is restored, there is an excessive increase in REM duration).
Why not the other options?
Impaired driving → More associated with total sleep deprivation , particularly lack of NREM Stage 3 (deep sleep) .
Teeth grinding (bruxism) → Occurs mostly in Stage 2 NREM sleep , not REM deprivation.
Increased appetite → More linked to chronic sleep deprivation affecting leptin and ghrelin levels , not specifically REM deprivation.
Sleep terrors → Occur in Stage 3 (slow-wave sleep, SWS) , not REM sleep.
Thus, the correct answer is impaired memory , as REM sleep is essential for memory consolidation and cognitive function .
Think about deep sleep disorders that all occur during Stage 3 SWS.
72 / 118
Category:
Neurosciences – Physiology
Which of the following is mostly associated with slow-wave sleep?
Slow-wave sleep (SWS) refers to deep sleep (Stages 3 and 4 of NREM sleep) and is characterized by:
Delta waves (low frequency, high amplitude) on EEG.
Decreased heart rate, respiratory rate, and muscle tone .
Consolidation of declarative memory .
Parasomnias (sleep terrors, sleepwalking, and night terrors) occur most commonly during Stage 3 SWS .
Sleep terrors (night terrors) are a type of parasomnia that occurs during deep NREM sleep (Stage 3) and are characterized by:
Sudden arousals with intense fear .
Autonomic hyperactivation (tachycardia, sweating, screaming, confusion) .
No memory of the event upon waking .
Why not the other options?
Irregular heart rate → More characteristic of REM sleep , where autonomic variability occurs.
Dreaming → Primarily occurs in REM sleep , not slow-wave sleep.
High-frequency EEG waves → Seen in wakefulness and REM sleep , while SWS is dominated by low-frequency delta waves .
Memory → SWS is associated with declarative memory consolidation , but sleep terrors are more directly linked to SWS.
Thus, sleep terrors are the best answer because they are most strongly associated with slow-wave sleep (Stage 3 NREM sleep) .
Think of the hypothalamic nucleus that controls sleep-wake cycles by regulating melatonin secretion .
73 / 118
Think about the hypothalamic center responsible for stopping food intake —its damage results in uncontrolled eating and weight gain .
74 / 118
Category:
NeuroSciences – Anatomy
A lesion in which of the following nuclei in the hypothalamus causes hypothalamic obesity?
The ventromedial nucleus (VMN) of the hypothalamus is the satiety center , meaning it signals the body to stop eating when energy needs are met. Damage to the ventromedial nucleus results in uncontrolled eating (hyperphagia) and obesity , a condition known as hypothalamic obesity .
The ventromedial nucleus is regulated by leptin , a hormone secreted by adipose tissue that inhibits hunger.
Lesions in the VMN disrupt leptin signaling , leading to excessive food intake and weight gain .
Why not the other options?
Lateral hypothalamic nucleus → The hunger center —lesions here cause anorexia and weight loss , not obesity.
Dorso-medial nucleus → Involved in behavioral and autonomic responses , but not directly linked to feeding control.
Supra-optic nucleus → Produces antidiuretic hormone (ADH) and regulates water balance, not feeding behavior.
Supra-chiasmatic nucleus → Controls circadian rhythms , not appetite regulation.
Thus, the ventromedial nucleus is the correct answer, as its lesion leads to hypothalamic obesity .
distinguishes between sensory ataxia (dorsal column damage) and cerebellar ataxia —only sensory ataxia improves with visual input.
75 / 118
Fine touch is carried by the dorsal column , while crude touch is carried by the anterior spinothalamic tract —which one is affected here?
76 / 118
Category:
NeuroSciences – Anatomy
A patient loses the sensation of crude touch, while the sensation of fine touch is intact. Which of the following tracts is most likely to be damaged?
The ventral (anterior) spinothalamic tract is responsible for transmitting crude touch and pressure sensations. Damage to this tract would result in loss of crude touch , while fine touch remains intact , as fine touch is conveyed by the dorsal column-medial lemniscus (DCML) pathway .
Sensory Tracts and Their Functions :
Ventral (anterior) spinothalamic tract → Crude touch and pressure
Lateral spinothalamic tract → Pain and temperature
Dorsal column tract (DCML pathway) → Fine touch, vibration, proprioception
Spinocerebellar tract → Unconscious proprioception (balance and coordination)
Spino-reticular tract → Pain perception and arousal response
Why not the other options?
Spinocerebellar tract → Involved in unconscious proprioception , not touch sensation.
Dorsal column tract → Carries fine touch, vibration, and proprioception , but fine touch is intact in this case.
Lateral spinothalamic tract → Conveys pain and temperature , which are not affected here.
Spino-reticular tract → Primarily involved in pain modulation and alertness , not touch sensation.
Thus, damage to the ventral (anterior) spinothalamic tract leads to loss of crude touch , making it the correct answer.
Think about the pathway responsible for fine touch, vibration, and proprioception.
77 / 118
Category:
NeuroSciences – Anatomy
Which of the following tract conveys information about highly localized touch sensations and proprioception?
The dorsal column-medial lemniscus (DCML) pathway is responsible for transmitting highly localized touch (fine touch), vibration, and proprioception from the body to the brain .
Pathway of the Dorsal Column Tract :
First-order neurons : Sensory signals from the skin and proprioceptors travel via peripheral nerves and enter the spinal cord through the dorsal root ganglion (DRG) .
Fibers from the lower body (legs) → ascend in the gracile fasciculus .
Fibers from the upper body (arms) → ascend in the cuneate fasciculus .
Second-order neurons : In the gracile and cuneate nuclei of the medulla, these neurons decussate (cross) as internal arcuate fibers and form the medial lemniscus .
Third-order neurons : From the ventral posterolateral (VPL) nucleus of the thalamus , fibers project to the primary somatosensory cortex (postcentral gyrus, Brodmann areas 3, 1, 2) for processing.
Why not the other options?
Spino-reticular tract → Involved in pain perception and alertness , not fine touch or proprioception.
Lateral spinothalamic tract → Carries pain and temperature sensations, not fine touch.
Spinocerebellar tract → Transmits unconscious proprioception to the cerebellum , not fine touch.
Anterior spinothalamic tract → Carries crude touch and pressure , not highly localized touch.
Thus, the dorsal column tract (DCML pathway) is the correct answer as it is responsible for fine touch and proprioception .
Think of the that nucleus as the “feeding center” —damage leads to starvation and weight loss .
78 / 118
Category:
NeuroSciences – Anatomy
A 20-year-old girl presented with severe weight loss. On history, she admitted that her appetite is greatly reduced and she is experiencing sleep disturbances for the last two months. Which hypothalamic nucleus controls the appetite center?
The lateral hypothalamic nucleus is responsible for stimulating hunger and appetite . It acts as the feeding center , and its activation promotes food intake .
Lesions in the lateral hypothalamus lead to anorexia and weight loss , which fits the patient’s symptoms of reduced appetite and severe weight loss .
The lateral hypothalamus is regulated by ghrelin (which stimulates hunger) and leptin (which inhibits hunger) .
Why not the other options?
Anterior hypothalamic nuclei → Involved in temperature regulation (cooling) and parasympathetic activity , not appetite.
Medial hypothalamic nuclei → No distinct “medial” hypothalamic nucleus exists as a primary appetite center. Likely referring to the ventromedial nucleus , which is the satiety center (opposite of the lateral nucleus). Lesions here cause hyperphagia (overeating), not weight loss .
Paraventricular nuclei → Involved in stress response and hormone regulation (secretes CRH, oxytocin, and ADH), not appetite.
Posterior hypothalamic nuclei → Controls heat production (thermoregulation) and sympathetic activity , not appetite regulation.
Thus, the lateral hypothalamic nucleus is the correct answer as it plays a key role in stimulating hunger and appetite .
A posterior cerebral artery (PCA) infarct affecting the primary visual cortex
79 / 118
Category:
NeuroSciences – Anatomy
A vascular lesion of the primary visual area (Brodmann area 17) may cause which one of the following defects?
The primary visual cortex (Brodmann area 17) is located in the occipital lobe , specifically in the calcarine sulcus . It receives visual information from the lateral geniculate nucleus (LGN) of the thalamus via the optic radiations .
A vascular lesion of the primary visual cortex , typically due to an infarct in the posterior cerebral artery (PCA) , results in contralateral homonymous hemianopia with macular sparing .
Macular sparing occurs because the macular representation in the occipital lobe receives dual blood supply from both the posterior cerebral artery (PCA) and the middle cerebral artery (MCA) .
Why not the other options?
Binasal hemianopia → Results from bilateral compression of the optic nerves at the optic chiasm , often due to internal carotid artery aneurysms , not a cortical lesion.
Blindness → Would occur with bilateral occipital lobe damage or bilateral optic nerve lesions , but a vascular lesion in one hemisphere would not cause total blindness.
Bitemporal hemianopia → Results from optic chiasm compression , typically due to a pituitary tumor , not a cortical lesion.
Contralateral homonymous hemianopia → This occurs with lesions of the optic tract, LGN, or optic radiations , but without macular sparing . A cortical lesion in Brodmann area 17 results in homonymous hemianopia with macular sparing .
Thus, contralateral homonymous hemianopia with macular sparing is the correct answer, as it is the hallmark of primary visual cortex lesions due to PCA infarcts .
The same vesicle that forms your cerebral cortex also builds the deep motor “command centers” just beneath it.
80 / 118
Category:
Neurosciences – Embryology
Basal nuclei are a collection of cell bodies that control voluntary movements. Which of the following vesicles gives rise to basal nuclei?
The basal nuclei (basal ganglia) are a collection of subcortical gray matter structures that play a key role in voluntary movement control, motor learning, and habit formation . These nuclei include:
Caudate nucleus
Putamen
Globus pallidus
Nucleus accumbens
Claustrum
The basal nuclei originate from the telencephalon , which is one of the five secondary brain vesicles derived from the prosencephalon (forebrain) .
Why not the other options?
Metencephalon → Forms the pons and cerebellum , not the basal nuclei.
Diencephalon → Forms the thalamus, hypothalamus, and subthalamus , but not the basal nuclei.
Mesencephalon → Forms the midbrain (including the substantia nigra, which is functionally related to the basal ganglia but not part of the basal nuclei themselves).
Myelencephalon → Develops into the medulla oblongata , unrelated to the basal nuclei.
Thus, the telencephalon is the correct answer as it gives rise to the basal nuclei along with the cerebral cortex .
Think about what structures are herniating —if only meninges and CSF are involved..
81 / 118
Think about neural tube defects —the anterior neuropore is responsible for forming the brain , while the posterior neuropore forms the spinal cord .
82 / 118
Category:
Neurosciences – Embryology
Meroanencephaly is defined as the partial absence of the brain. It is caused by which of the following defects?
Meroanencephaly (a form of anencephaly ) results from the failure of the anterior neuropore to close during neurulation (approximately day 25 of embryonic development ). This leads to partial absence of the brain and skull , often exposing neural tissue to amniotic fluid, causing degeneration.
Neural Tube Development and Closure Defects:
Anterior neuropore (rostral end) fails to close → Leads to meroanencephaly/anencephaly (absence of parts of the brain and skull).
Posterior neuropore (caudal end) fails to close → Leads to spina bifida (defects in the vertebral column and spinal cord).
Why not the other options?
Defect in regionalization → Affects brain patterning (e.g., holoprosencephaly), but not neural tube closure.
Disorder of myelination → Affects nerve function (e.g., multiple sclerosis, leukodystrophies), not brain formation.
Failure of neural crest cell formation → Leads to defects in craniofacial structures, peripheral nerves, and ganglia (e.g., Hirschsprung disease, Treacher Collins syndrome), but not anencephaly.
Failure of closure of posterior neuropore → Causes spina bifida , not anencephaly.
Thus, meroanencephaly occurs due to the failure of anterior neuropore closure , preventing proper brain formation.
Think of the autonomic system that promotes digestion, relaxation, and energy conservation
83 / 118
Category:
Neurosciences – Physiology
During resting, the heart rate is slowed, pupils are constricted, peristalsis and glandular activity are increased, sphincters are opened, and the bladder wall is contracted. Which of the following system is activated in this state?
The physiological effects described—slowing of heart rate, pupil constriction, increased peristalsis and glandular activity, sphincter relaxation, and bladder contraction —are all characteristic of the parasympathetic nervous system (PNS) .
The PNS , also known as the “rest and digest” system , is responsible for conserving energy and promoting digestion and excretion . It primarily functions through the vagus nerve (CN X) and sacral spinal nerves.
Key Parasympathetic Effects:
Cardiovascular system → Decreases heart rate (bradycardia) by inhibiting the SA node via the vagus nerve .
Eyes → Pupil constriction (miosis) via activation of the Edinger-Westphal nucleus (oculomotor nerve, CN III).
Gastrointestinal tract → Increased peristalsis and glandular secretion , promoting digestion.
Urinary system → Bladder contraction (detrusor muscle) and sphincter relaxation , facilitating urination.
Why not the other options?
Sympathetic nervous system → Would cause increased heart rate, pupil dilation (mydriasis), decreased peristalsis, and bladder relaxation (opposite effects).
Enteric nervous system → Regulates the GI tract but does not control heart rate or pupil size.
Somatic nervous system → Controls voluntary muscle movements, not autonomic functions.
Think about the spinocerebellar pathways —the cuneocerebellar tract is specifically for the upper body , just like the dorsal spinocerebellar tract is for the lower body .
84 / 118
Think about the three-layered structure of the cerebellar cortex and the types of inhibitory interneurons present in each layer.
85 / 118
Category:
NeuroSciences – Anatomy
Which of the following is the correct option regarding the cerebellum?
The cerebellar cortex has three layers :
Molecular layer (outermost) → Contains stellate and basket cells , along with Purkinje dendrites and parallel fibers.
Purkinje cell layer (middle) → Contains Purkinje cells , which are the primary output neurons of the cerebellar cortex .
Granular layer (innermost) → Contains granule cells and Golgi cells .
Why not the other options?
Purkinje neurons are of bipolar variety → Incorrect; Purkinje cells are large, inhibitory (GABAergic) neurons with extensively branched dendritic trees (not bipolar).
Superior cerebellar peduncle connects cerebellum to pons → Incorrect; the superior cerebellar peduncle connects the cerebellum to the midbrain , not the pons. The middle cerebellar peduncle connects it to the pons.
Molecular, granular, and pyramidal layers are present in cortex → Incorrect; the cerebral cortex has a pyramidal layer, but the cerebellar cortex does not . Instead, it has a molecular layer, Purkinje cell layer, and granular layer .
Purkinje cells have scarce branching → Incorrect; Purkinje cells have highly branched dendrites , forming one of the most complex dendritic trees in the CNS.
Thus, the correct answer is that stellate and basket cells are present in the molecular layer .
Think of the ciliated cells that help circulate CSF in the ventricles and spinal cord .
86 / 118
The genu of the internal capsule carries motor fibers to cranial nerves , which are crucial for speech articulation .
87 / 118
Category:
NeuroSciences – Anatomy
Which of the following can be caused by an infarction of the genu of the internal capsule?
The genu of the internal capsule primarily contains the corticobulbar tract , which carries motor signals from the cortex to the cranial nerve nuclei controlling the muscles of the face, tongue, pharynx, and larynx . An infarct in the genu can result in dysarthria (difficulty in articulating speech) due to weakness in the orofacial and laryngeal muscles .
Why not the other options?
Visual impairment → Associated with optic radiation lesions (in the posterior limb of the internal capsule or occipital lobe).
Agitation → More related to frontal lobe or limbic system damage , not the genu of the internal capsule.
Confusion → Typically linked to diffuse cortical dysfunction , not a localized infarct in the internal capsule.
Auditory deficits → Result from damage to the auditory pathway (medial geniculate body, auditory cortex, or brainstem structures) , not the internal capsule.
Thus, an infarct of the genu of the internal capsule most commonly leads to dysarthria due to corticobulbar tract involvement.
Gray rami communicantes carry postganglionic sympathetic fibers from the sympathetic chain to spinal nerves.
88 / 118
Think about the limbic structure that curves downward in front of the interventricular foramen .
89 / 118
Think about the midbrain structure responsible for auditory reflexes and signal integration .
90 / 118
Think of small penetrating arteries that supply the deep structures of the brain —the lenticulostriate arteries are key.
91 / 118
Think of lateral medullary syndrome (Wallenberg syndrome) , which is caused by an infarct in PICA , affecting the medulla .
92 / 118
Think of olfactory signals crossing between hemispheres
93 / 118
Think about non-fluent aphasia , where patients understand language but struggle to form words .
94 / 118
Consider which cranial nerve exits through the stylomastoid foramen and what functions it controls.
95 / 118
Think about the jugular foramen , which transmits CN IX, X, and XI —it is located in the posterior cranial fossa , primarily formed by the occipital bone .
96 / 118
Category:
NeuroSciences – Anatomy
A radiologist, while reading the computed tomography scan, noticed a jugular tubercle grooved by IX, X, and XI cranial nerves. This tubercle is part of which of the following bones?
The jugular tubercle is a bony prominence located on the occipital bone , near the jugular foramen . The jugular foramen is the passage through which cranial nerves IX (Glossopharyngeal), X (Vagus), and XI (Accessory) exit the skull.
The temporal bone contributes to the formation of the jugular foramen but does not contain the jugular tubercle.
The ethmoid bone is located anteriorly and forms part of the nasal cavity and the cribriform plate (where the olfactory nerve (CN I) passes).
The parietal bone is part of the skull vault and does not contain major foramina for cranial nerves.
The sphenoid bone houses important foramina like the optic canal (CN II) and the superior orbital fissure (CN III, IV, V1, VI) , but it does not form the jugular tubercle.
Thus, the occipital bone is the correct answer.
Cerebellar lesions cause ipsilateral limb ataxia —look at which side the coordination is impaired.
97 / 118
Category:
NeuroSciences – Anatomy
A 60-year-old hypertensive male was taken to the emergency department with complaints of sudden onset of nausea and vomiting. The examination revealed slurred speech and dysmetria on finger-to-nose test on the left side. However, his gait was normal. In which of the following areas lesion has occurred?
The patient presents with sudden-onset nausea, vomiting, slurred speech, and dysmetria (impaired coordination), which are hallmark signs of a cerebellar lesion .
The cerebellar hemispheres control coordination of the ipsilateral limbs , meaning a lesion in the left cerebellar hemisphere would result in left-sided dysmetria (as seen in the finger-to-nose test).
The flocculonodular lobe and vestibular system are primarily responsible for balance and eye movements . Since the patient’s gait is normal , a vestibular-related lesion is unlikely.
The fastigial nucleus is involved in truncal stability and balance, but no truncal ataxia is reported.
A right cerebellar lesion would cause right-sided symptoms, which does not match the findings in this patient.
Thus, the most likely affected area is the left cerebellar hemisphere .
Think about the thalamic relay station for sensory information coming from the face and trigeminal nerve .
98 / 118
Think about which glial cells have an immune function —these do not arise from the same germ layer as neurons.
99 / 118
Think about the embryonic structure responsible for sensory processing in the spinal cord.
100 / 118
Think of the basal ganglia structure that controls movement inhibition and how its damage leads to uncontrolled flinging movements .
101 / 118
Think of the dopamine-producing structure in the midbrain that degenerates in Parkinson’s disease.
102 / 118
Think about which ion is most responsible for depolarizing the postsynaptic neuron in neuromuscular junctions and autonomic ganglia .
103 / 118
Focus on the neurotransmitter that is not a catecholamine and is instead linked to mood regulation and sleep .
104 / 118
Consider psychiatric disorders where dopamine-blocking drugs (antipsychotics) are the main treatment.
105 / 118
Category:
Neurosciences – Pathology
Which of the following is associated with increased dopamine levels?
Schizophrenia is strongly linked to increased dopamine activity , particularly in the mesolimbic pathway , which is associated with positive symptoms such as hallucinations and delusions. The dopamine hypothesis of schizophrenia suggests that hyperactivity of dopamine transmission, especially at D2 receptors , contributes to the disorder.
Global aphasia without hemiparesis is a language disorder typically caused by left hemisphere damage, not related to dopamine dysfunction.
Parkinson’s disease is caused by dopamine deficiency due to loss of dopaminergic neurons in the substantia nigra .
Agnosia (the inability to recognize objects, sounds, or sensations) is linked to cortical dysfunction, particularly in the temporal or parietal lobes, not dopamine imbalance.
Alzheimer’s disease is associated with decreased acetylcholine , not dopamine excess.
Think about signaling molecules made of small chains of amino acids that often act slower than classical neurotransmitters but have longer-lasting effects , especially in pain modulation and emotional responses.
106 / 118
Look for a fiber that connects different parts within the same hemisphere , rather than crossing to the other hemisphere or linking to lower brain structures.
107 / 118
Consider which brain wave pattern is dominant when someone is actively thinking and engaging in a task .
108 / 118
Think about which cells in the substantia nigra are primarily affected in Parkinson’s disease and what neurotransmitter they produce.
109 / 118
Think about how water appears in T2-weighted MRI—structures with high water content show up as hyperintense
110 / 118
Think of the hypothalamic nucleus that, when stimulated, drives a person to eat and drink.
111 / 118
Think about the catecholamine pathway and the neurotransmitter that is synthesized early in this process.
112 / 118
Consider the supportive glial cells that help regulate the brain’s microenvironment and contribute to the selective permeability of the BBB.
113 / 118
Think about chronic infections that cause fibrin deposition in the CSF, rather than acute inflammatory responses.
114 / 118
Think about the essential amino acid that serves as a precursor for a neurotransmitter involved in mood regulation.
115 / 118
Consider conditions that affect the neuromuscular junction and how autoimmune processes could interfere with signal transmission.
116 / 118
Think about how venous blood from the superior part of the brain ultimately reaches the internal jugular vein.
117 / 118
Think about what type of sensations are carried by the dorsal column and how their loss might affect body coordination.
118 / 118
Category:
Neurosciences – Physiology
Lesion of the dorsal column results in which of the following?
The dorsal column is responsible for transmitting fine touch, vibration, and proprioceptive (position sense) information from the periphery to the brain. A lesion here disrupts proprioceptive feedback, leading to difficulty in coordination and balance. This manifests as sensory ataxia, which occurs due to impaired sensory input, rather than motor system dysfunction.
Your score is
The average score is 51%
Restart quiz
Thank you for your feedback.